You are on page 1of 229

Expert Panel on

Neuroradiology 2013

All questions and images that appear in this module were authored and
critically reviewed by a panel of experts in the field of neuroradiology to
ensure that they were functional and valid. The experts reviewed each
question and image in terms of its applicability to a general radiologist’s
practice today. In addition, they assessed the timeliness of the content and
the quality of the images to determine whether they met today’s quality
standards. The expert panel also provided the discussions for each answer
published in this report as well as the references of educational sources for
further learning. We wish to thank the panel members for their expert input
and review.
Nafi Aygun, MD, Chair
Asim F. Choudhri, MD
Steven A. Messina, MD
Rohini N. Nadgir, MD
Anna E. Nidecker, MD
 
Nafi Aygun, MD
CPI Neuroradiology Panel Chair/Editor
Associate Professor of Radiology, Director, Neuroradiology Fellowship
Program Johns Hopkins University
 
Harris L. Cohen, MD, FACR
CPI Program Chair/Editor in Chief
Chairman of Radiology, University of Tennessee Health Science Center;
Radiologist-in-Chief, Le Bonheur Children’s Hospital
 
Donald B. Price, MD
Assistant Editor in Review
Associate Professor of Clinical Radiology,
Stony Brook University School
of Medicine; Chief, Neuroradiology Division,
Winthrop University Hospital
 
Matthew T. Whitehead, MD
Assistant Editor in Review
Assistant Professor, Neuroradiology, Children’s National Medical Center
Objectives
This self-assessment module (SAM) is a component of the American
College of Radiology’s Continuous Professional Improvement (CPI)
program. The intent of the CPI program is to assess your overall imaging
knowledge including areas evaluated by the American Board of Radiology’s
oral examination and to provide you with information on the items tested.
The SAM objectives for CPI Neuroradiology Module 2013 are as follows:
• Assess and manage a variety of hypothetical clinical
situations related to neuroradiology;
• Assess image findings and exercise independent
medical judgment based on the images and/or facts
provided concerning neuroradiology;
• Identify clinical and imaging principles relevant to
the cases encountered and topics discussed in the
practice of neuroradiology; and
• Enhance clinical practice strategies based on the
practitioner’s self-assessment in the area of
neuroradiology.
CPI FORMAT: To provide you with an improved self-assessment activity,
CPI modules are published in a combined module/report format. This
format allows you immediate feedback and expedites the processing of
claiming continuing medical education credits.
You should begin by answering the 50 multiple-choice questions. This
activity should take approximately 8 hours. Record your answers on the
answer sheet provided.
On completion of the exam, refer to the Answer Key located after the
question section to determine your score. Rationales and references for each
question are printed following the Answer Key. As soon as possible after
grading the test, focus on the questions you missed and reinforce your
learning.
When you have calculated your score, follow the instructions below to log
on, record your score, and claim CME and/or SAM credit.
1. Calculate Your Test Score.
Please use the answer sheet provided to record your answers. Grade your
test using the Answer Key located after the question section of the module.
2. Claim Your Credit Online.
A. Go to the ACR Education page by typing
http://www.acr.org/education into your address bar.
Click on My Education Page and log in.
B. Click the My Activities tab and when the list opens,
find CPI Neuroradiology Module 2013 and click
Launch. This will open a new window. If you do not see
the Instructions page, you may need to disable the
pop-up blocker.
C. To complete the claim process, it is required that
each section listed on the left-hand menu be opened.
• Fill in Your Scores by entering the total number
of questions you answered correctly from this
self-assessment module.
• Answer the Required Posttest questions from
the module for SAM credits.
• View the References,.
• Complete the Required Program Evaluation.
• View the Score Comparison Graphs to learn
how your scores compare to those of other CPI
self-assessment participants.
3. Print Your Certificate.
After you complete the Required Program Evaluation, close the pop-up
window and click the Completed tab. Your credits for CPI Neuroradiology
Module 2013 will be recorded, and you may access your CME and SAM
certificate by clicking Certificate.
Accreditation
The American College of Radiology (ACR) is accredited by the
Accreditation Council for Continuing Medical Education (ACCME) to
provide continuing medical education for physicians.
The ACR designates this educational activity for 8 AMA PRA Category 1
Credits™. Physicians should only claim credit commensurate with the
extent of their participation in the activity. This module carries valid CME
for three years from the publication release date of August 2013.
CPI Neuroradiology Module 2013 is qualified for 5 SAM credits by the
American Board of Radiology in meeting the criteria for self-assessment
toward the purpose of fulfilling requirements in the ABR Maintenance of
Certification Program.
The ACR has received no commercial support for this activity. The faculty
listed below have indicated that they have no relevant financial relationships
related to the presentation of this material. They also do not intend to
discuss the use of a medical device or pharmaceutical that is classified by
the FDA as investigational for the intended use, or that is “off-label,” eg, a
use not described on the product’s label.
Nafi Aygun, MD
Asim F. Choudhri, MD
Steven A. Messina, MD
Harris L. Cohen, MD, FACR
Rohini N. Nadgir, MD
Anna E. Nidecker, MD
Donald B. Price, MD
Matthew T. Whitehead, MD
Privacy Information: The ACR is committed to protecting your privacy and will collect no personal information from you unless

you choose to provide that information. If you purchase an ACR product online, we collect personal information required to

support the purchase of products, including names, addresses, and e-mail addresses. When existing demographic information in

our database is incorrect or incomplete, that information will be updated. The ACR only shares your personal information when

necessary to complete your order (eg, your credit card and billing information is shared with our bank for authorization, and your

delivery address is shared with our fulfillment center and shipper). The fulfillment center uses your e-mail address to send you an

order confirmation and to alert you to any changes in the availability of the items you ordered. The ACR sends e-mails to let you

know about changes to its Web site and to communicate with you about issues that matter to the professions of radiology, radiation

oncology, and medical physics, but we do not sell e-mail addresses to third parties. The ACR, like most associations, does sell

address information as a means of generating non-dues income. You may opt out of the e-mail contact or address label sales by

notifying the College’s Membership Services Department at (800) 347-7748 or membership@acr.org.

Release Date: 08/2013 Expiration Date: 08/2016


 
Self-Assessment Examination
CPI Neuroradiology Module 2013

IMAGE-RELATED QUESTIONS
 
1. A 52-year-old man presents with 3 hours of left-sided
weakness. Upon review of the findings in Figure 1-1,
noncontrast computed tomography (CT), and Figures 1-
2, 1-3, 1-4, 1-5, 1-6, and 1-7, brain magnetic resonance
(MR) images, which one of the following statements is
TRUE?
A. Intravenous tissue plasminogen activator (tPA) is
contraindicated.
B. Intra-arterial tPA/thrombolysis is contraindicated.
C. Neither intravenous nor intra-arterial
tPA/thrombolysis is contraindicated.
D. None of the above statements are true.

Review Answer 1
 
2. A 68-year-old man presents with a history of a
hypertensive cerebral bleed with substantial
intraventricular extension. Figure 2-1, a noncontrast
CT image, and Figures 2-2 and 2-3, CT angiography
(CTA) images, were obtained. A subsequent CT image
(Figure 2-4) performed the day after ventriculostomy
placement demonstrated marked expansion of the
intraventricular component of the hemorrhage. Which
one of the following findings on the images MOST
likely predicts that the hematoma will increase in size?
A. Complete casting of the right lateral ventricle on the
prior noncontrast-enhanced CT and CTA
B. Hemorrhage within the left lateral ventricle
C. Midline shift
D. Punctate enhancement within the hematoma

Review Answer 2
 
3. A 67-year-old man presents with sudden loss of
consciousness. Based on findings on the CT images,
Figures 3-1 and 3-2, which one of the following is the
MOST likely diagnosis?
A. Aneurysmal rupture
B. Hypertensive hemorrhage
C. Amyloid angiopathy
D. Arteriovenous malformation rupture

Review Answer 3
 
4. You are shown axial images from a noncontrast CT scan
(Figure 4-1) and from a repeat scan performed the
following morning (Figure 4-2). Which one of the
following choices is the MOST likely reason for the left
perisylvian hyperdensity seen on the noncontrast CT?
A. Intra-axial hematoma
B. Extra-axial hemorrhage
C. Acute cortical laminar necrosis
D. Extravasation of contrast agent administered during
thrombolysis

Review Answer 4
 
5. A 42-year-old patient presents to a neurologist with
headache and symptoms worrisome for early onset
dementia. Figures 5-1, 5-2, 5-3, and 5-4 are selected
FLAIR images. Which one of the following disease
process etiologies is MOST likely implicated by the
findings on these images?
A. Prion disease
B. Antibodies to aquaporin 4
C. Severe bilateral internal carotid artery stenoses
D. Genetic mutation (NOTCH3)

Review Answer 5
 
6. Based on the susceptibility-weighted image (Figure 6-
1), which one of the following differential
considerations would be LEAST likely for this patient?
A. Intracranial hypotension
B. Myxopapillary ependymoma
C. Prior cerebral aneurysm rupture
D. Cerebral amyloid angiopathy
E. Arteriovenous malformation

Review Answer 6
 
7. Which one of the findings in Figure 7-1 is MOST likely
related to treatment of the patient’s condition?
A. Serpentine right lenticulostriate vessels
B. Small caliber of right M2 and M3 branches
C. Prominence of the right superficial temporal artery
D. Nondilated ventricular system

Review Answer 7
 
8. A patient presents with peripartum hypertension.
Based on Figures 8-1, 8-2, 8-3, and 8-4, which one of
the following is the MOST likely diagnosis?
A. Acute multifocal infarcts
B. Multiple sclerosis
C. Posterior reversible encephalopathy syndrome
D. Periventricular leukomalacia

Review Answer 8
 
9. A patient presents with acute confusion, weakness, and
fever. Based on Figures 9-1, 9-2, 9-3, 9-4, and 9-5,
which one of the following is the MOST likely
diagnosis?
A. High-grade glioma
B. Abscess
C. Acute infarct
D. Metastasis

Review Answer 9
 
10. Based on the findings in Figures 10-1, 10-2, 10-3, 10-4,
and 10-5, which one of the following organisms is the
MOST likely cause for the imaging appearances?
A. Herpes simplex virus
B. Tuberculosis
C. Human immunodeficiency virus
D. Listeria

Review Answer 10
 
11. An axial T2-weighted image (Figure 11-1) shows
bilateral hyperintense signal in the globi pallidi. Which
one of the following mechanisms would MOST likely
result in this pattern of injury?
A. Near drowning
B. Moyamoya disease
C. Carbon monoxide toxicity
D. Diffuse axonal injury
E. Cardioembolic stroke

Review Answer 11
 
12. A patient presents with bilateral sensory deficits.
Based on the images (Figures 12-1, 12-2, 12-3, and 12-
4), which one of the following is the MOST likely
diagnosis?
A. Metastatic disease
B. Posterior reversible encephalopathy syndrome
C. Septic emboli
D. Demyelinating disease (multiple sclerosis)

Review Answer 12
 
13. A 57-year-old woman presents with progressive
balance problems noted while walking. She has fallen 5
times in the last 6 months. Based on her MRI (Figure
13-1), which one of the following is NOT a
characteristic feature of her condition?
A. Labile mood
B. Blurring of vision and poor control of eye movement
C. Slurred speech
D. Choreiform movements

Review Answer 13
 
14. A 60-year-old woman presents with headaches. Which
one of the following statements is MOST correct
regarding the lesion identified on Figures 14-1, 14-2,
14-3, 14-4 and 14-5?
A. Central calcifications can be seen on CT.
B. The abnormality occurs more often in men than in
women.
C. There is usually a good prognosis, typically with
resolution following resection.
D. There are high recurrence rates even after surgery
and/or radiation therapy.

Review Answer 14
 
15. A 21-year-old woman presents with seizures. Which
one of the following is the MOST likely etiology of the
lesion seen on imaging (Figures 15-1 and 15-2)?
A. Primitive neuroectodermal tumor
B. Mesial temporal sclerosis
C. Dysembryoplastic neuroepithelial tumor
D. Pilocytic astrocytoma
E. Anaplastic astrocytoma

Review Answer 15
 
16. You are shown MR images (Figures 16-1, 16-2, and 16-
3) of a 16-year-old patient with a history of epilepsy.
Which one of the following is the MOST likely
diagnosis?
A. Encephalitis
B. Tumor
C. Developmental anomaly
D. Vascular malformation

Review Answer 16
 
17. A patient presents with hearing loss. Based on the CT
images of the temporal bone (Figures 17-1 and 17-2),
which one of the following is the MOST likely
diagnosis?
A. Otospongiosis
B. Tympanosclerosis
C. Acquired cholesteatoma
D. Paget disease

Review Answer 17
 
18. A 49-year-old woman presents to the emergency
department with 5 years of left-sided tinnitus,
intermittent left ear pain, and occasional dizziness. She
has a known left sphenoid meningioma. The lesion
identified in the images (Figures 18-1, 18-2, 18-3, and
18-4) is MOST consistent with which one of the
following diagnoses?
A. Glomus jugulare
B. Jugular bulb thrombosis
C. Carotid dissection
D. Schwannoma

Review Answer 18
 
19. A 38-year-old man presents with a history of
headaches. CT and MR images are obtained (Figures
19-1, 19-2, 19-3, and 19-4). Which one of the following
features identified in the images MOST strongly
supports a diagnosis of an inflammatory lesion rather
than a neoplastic process?
A. Homogeneous hyperintensity on T2-weighted
imaging
B. Hyperintensity on T1-weighted imaging
C. Lack of enhancement within the lesion
D. Bone expansion

Review Answer 19
 
20. A 45-year-old woman presents with chronic mild
occipital headache. MR images are obtained (Figures
20-1, 20-2, 20-3, and 20-4). Based on the signal
characteristics and morphology, which one of the
following is the MOST likely diagnosis?
A. Metastatic breast cancer
B. Chordoma
C. Cholesterol granuloma
D. Pituitary macroadenoma

Review Answer 20
 
21. A 40-year-old woman presents with painless swelling
in her right cheek. MR imaging is obtained (Figures 21-
1, 21-2, and 21-3). Which one of the following would
MOST likely be considered a feature of this lesion?
A. Poorly defined margins within the gland on imaging
B. Bilateral occurrence in as many as one-third of cases
C. Occasionally demonstrates malignant degeneration
D. Associated with human immunodeficiency virus
infection
E. Moderately elevated signal on T2-weighted imaging

Review Answer 21
 
22. A 59-year-old man presents with acute bilateral lower
extremity weakness. Based on selected images from a
thoracic spine MRI (Figures 22-1, 22-2, 22-3, 22-4, and
22-5), which one of the following is the MOST likely
diagnosis?
A. Multiple sclerosis
B. Neuromyelitis optica
C. Ischemia
D. Astrocytoma

Review Answer 22
 
23. A young patient presents with back pain. Based on the
images provided (Figures 23-1, 23-2, 23-3, and 23-4),
which one of the following is the MOST likely
diagnosis?
A. Discitis/osteomyelitis
B. Scheuermann disease
C. Metastasis
D. Ankylosing spondylitis

Review Answer 23
 
24. A 57-year-old man with a history of bladder cancer
presents with back pain. Based on the images provided
(Figures 24-1, 24-2, 24-3, and 24-4), which one of the
following is the MOST likely diagnosis?
A. Hemangiomas
B. Paget disease
C. Lytic metastases
D. Osteoblastic metastasis

Review Answer 24
 
25. A 32-year-old afebrile woman presents to the
emergency department with a stiff neck. She
undergoes a contrast-enhanced CT (Figures 25-1 and
25-2) of the neck. Upon notifying the referring
physician of findings from the sagittal plane
multiplanar reformatted image (Figure 25-1), which
one of the following is the referring physician’s MOST
likely next step?
A. Administer antibiotics and consult with
otolaryngology and/or neurosurgery for possible
drainage
B. Request a cervical spine MRI
C. Perform a lumbar puncture
D. Administer nonsteroidal anti-inflammatory drugs and
arrange for clinical follow-up

Review Answer 25
NONIMAGE-RELATED QUESTIONS
 
26. Which intracerebral blood product is MOST likely
hyperintense on gradient-echo and susceptibility-
weighted imaging?
A. Hemosiderin
B. Deoxyhemoglobin
C. Intracellular methemoglobin
D. Extracellular methemoglobin

Review Answer 26

 
27. Which one of the following MR sequences is MOST
sensitive for the detection of parenchymal
microhemorrhages?
A. Diffusion-weighted imaging
B. Susceptibility-weighted imaging
C. Gradient-echo
D. Fluid-attenuated inversion-recovery

Review Answer 27

 
28. A 70-year-old man presents with a large parietal lobe
hemorrhage. He has no significant medical conditions
and is normotensive on exam. On a gradient-echo
sequence brain MRI, innumerable small foci of
susceptibility indicating microhemorrhages are
identified in a subcortical distribution. Which one of
the following is the MOST likely cause for his parietal
lobe hemorrhage?
A. Hypertension
B. Primary brain tumor
C. Amyloid angiopathy
D. Trauma

Review Answer 28

 
29. Which one of these findings on a noncontrast-
enhanced head CT has NOT been described as an early
indicator of a middle cerebral artery occlusion and
resultant ischemia?
A. Hyperdense vessel
B. Blurring of the gray-white conspicuity of the insular
ribbon
C. Thalamic hypoattenuation
D. Basal ganglia hypoattenuation

Review Answer 29

 
30. A 58-year-old woman with no history of prior trauma
or vascular abnormality presents with a 3-week history
of redness in her left eye, mild proptosis, and an orbital
bruit heard on auscultation. Such features MOST likely
demonstrate which one of the following findings?
A. A fistula between the internal carotid artery (ICA)
and ophthalmic artery
B. A fistula between the cavernous segment of the ICA
and the cavernous sinus
C. A fistula between the dural branches of the external
carotid artery (ECA) and the cavernous sinus
D. A fistula between the dural branches of the ICA and
ECA

Review Answer 30

 
31. Which one of the following features is NOT typically
demonstrated with incidentally discovered isolated
cavernous malformations?
A. Hyperintensity on T2-weighted imaging
B. Mass effect and surrounding edema
C. Enhancement
D. Peripheral rim of hemosiderin deposition

Review Answer 31

 
32. Which one of the following is NOT a typical
therapeutic means of decreasing flow through an
arteriovenous malformation nidus?
A. Particle embolization
B. Glue embolization
C. Radiosurgery
D. Open surgery
E. Balloon occlusion

Review Answer 32

 
33. Which one of the following is the MOST likely location
for venous stenoses in patients with pseudotumor
cerebri?
A. Superior sagittal sinus
B. Transverse sinus
C. Straight sinus
D. Cavernous sinus
E. Internal jugular veins

Review Answer 33

 
34. Which one of the following findings has NOT been
associated with the diagnosis of intracranial
hypotension?
A. Dilation of dural venous sinuses
B. Dilation of optic nerve sheaths
C. Sagging of the mid brain below the plane of the
tentorial incisura
D. Tonsillar herniation
E. Subdural hematoma

Review Answer 34

 
35. Which one of the following conditions does NOT
typically show a basilar meningeal pattern of
enhancement on MRI?
A. Tuberculous meningitis
B. Intracranial hypotension
C. Neurosarcoid
D. Lymphoma

Review Answer 35

 
36. Which one of the following imaging findings is NOT
consistent with a diagnosis of herpes simplex virus
encephalitis?
A. Increased FLAIR signal in the mesial temporal lobe
B. Increased FLAIR signal along the cingulate gyrus
C. Increased FLAIR signal in the globus pallidus
D. Restricted diffusion in affected regions

Review Answer 36

 
37. Which one of the following types of therapy is MOST
associated with the “pseudoprogression” phenomenon
described on MRI in patients with high-grade primary
brain tumors?
A. Antiangiogenesis drugs such as bevacizumab
B. Radiation and chemotherapy with agents such as
temozolomide
C. Intrathecal methotrexate
D. Aggressive steroid treatment

Review Answer 37

 
38. Which one of the following imaging features MOST
strongly supports the diagnosis of primary central
nervous system lymphoma as opposed to glioblastoma
in a patient with a solitary brain mass?
A. Periventricular location
B. Restricted diffusion
C. Avid enhancement
D. Normal relative cerebral blood volume
E. Subependymal spread

Review Answer 38

 
39. Which ONE of the following findings suggests a non-
Alzheimer type of dementia?
A. Decreased metabolism in the temporal and parietal
lobes on fluorodeoxyglucose (FDG)-positron emission
tomography (PET) imaging
B. Disproportionate atrophy of the superior temporal
gyrus and the hippocampus
C. Marked ventriculomegaly that is out of proportion to
generalized brain atrophy
D. Increased binding of amyloid-labeled PET agent
florbetapir F 18

Review Answer 39

 
40. Which one of the following areas is LEAST likely to be
involved in Wernicke encephalopathy?
A. Thalami
B. Corpus callosum
C. Periaqueductal gray matter
D. Mamillary bodies
E. Cerebellar vermis

Review Answer 40

 
41. Mesial temporal sclerosis is bilateral in approximately
what percent of patients with the condition?
A. 1%
B. 15%
C. 50%
D. 85%
E. 99%

Review Answer 41

 
42. Which one of the following imaging findings is LEAST
likely to be associated with a Rathke cleft cyst?
A. Hyperdensity on CT
B. T1 shortening
C. T2 prolongation
D. Peripheral enhancement

Review Answer 42

 
43. Which one of the following nerve-deficit pairings is
INCORRECT?
A. Cranial nerve III: pupillary dilation
B. Cranial nerve II: decreased visual acuity
C. Cranial nerve VI: medial gaze palsy
D. Cranial nerve IV: double vision
E. Cranial nerve III: upward gaze palsy

Review Answer 43

 
44. Which one of the following conditions will NOT
present with pulsatile tinnitus?
A. Otosclerosis
B. Large vestibular aqueduct syndrome
C. Paget disease involving the skull base
D. Dural arteriovenous fistula

Review Answer 44

 
45. In the case of an intramural hematoma associated with
carotid or vertebral artery dissection, which one of the
following is the MOST appropriate length of time to
wait after the dissection to best see the increased T1
signal in the arterial wall?
A. 2 hours
B. 6 hours
C. 12 hours
D. 24 hours
E. 72 hours

Review Answer 45

 
46. In regards to human papilloma virus (HPV)-positive
and HPV-negative oropharyngeal carcinomas, which
one of the following statements is FALSE?
A. HPV-positive tumors are more radiosensitive and
carry a more favorable prognosis.
B. HPV-positive nodal metastases can demonstrate a
cystic configuration.
C. HPV-positive tumors more often present in older
patients.
D. HPV-positive tumors are more common in patients
with no history of tobacco use.

Review Answer 46

 
47. A young man presents with a several-week history of
numbness and paresthesias in his hands and feet, and
recently noticed some unsteadiness in his gait as well
as weakness. He is found to have megaloblastic
anemia. Which one of the following would MOST likely
be seen on a spine MRI?
A. Hyperintensity in the ventral horns on T2-weighted
imaging
B. Hyperintensity in the dorsal aspect of the spinal cord
on T2-weighted imaging
C. Enlargement of the central spinal canal
D. Diffuse thinning of the spinal cord

Review Answer 47

 
48. Which one of the following cervical spine injuries is
MOST likely a stable fracture?
A. Clay shoveler’s fracture
B. Flexion teardrop fracture
C. Isolated articular pillar
D. Hangman’s fracture

Review Answer 48
 
49. Which one of the following MRI findings is MOST
helpful to distinguish discitis/osteomyelitis from
degenerative changes?
A. Hyperintensity on T2-weighted imaging in disc
B. Endplate marrow edema
C. Perivertebral soft tissue edema
D. Loss of disc heigh

Review Answer 49

 
50. Which one of the following is the CORRECT
progression of marrow pathophysiology corresponding
to Modic types I, II, and III degenerative end plate
changes on MR?
A. Edema to fatty changes to sclerosis
B. Fatty changes to sclerosis to edema
C. Sclerosis to fatty changes to edema
D. Fatty changes to edema to sclerosis

Review Answer 50

 
END OF QUESTION SECTION
ANSWER KEY
CPI NEURORADIOLOGY
MODULE 2013
 
1. C
2. D
3. A
4. D
5. D
6. A
7. C
8. C
9. B
10. B
11. C
12. D
13. D
14. C
15. C
16. C
17. A
18. A
19. C
20. B
21. C
22. C
23. D
24. B
25. D
26. D
27. B
28. C
29. C
30. C
31. B
32. A
33. B
34. B
35. B
36. C
37. B
38. D
39. C
40. B
41. B
42. D
43. C
44. B
45. E
46. C
47. B
48. A
49. C
50. A
RATIONALES AND
REFERENCES

Answer 1 is C.
The test images demonstrate numerous secondary findings of middle
cerebral artery (MCA) occlusion. The noncontrast CT (Figure 1-1)
demonstrates a hyperdense right MCA indicating intraluminal thrombus
resulting in increased T2 signal on fluid-attenuated inversion-recovery
(FLAIR) imaging (Figure 1-2) in the MCA branches due to slow flow.
Increased signal on diffusion-weighted imaging (DWI) (Figure 1-3) and
decreased signal on apparent diffusion coefficient (ADC) map imaging
(Figure 1-4) signify restricted diffusion in the right basal ganglia compatible
with acute infarction. This involves less than one-third of the territory
supplied by the MCA, which is compatible with less than 70 ml of
parenchymal involvement. The area of restricted diffusion is considered the
core infarction. Furthermore, the T2 FLAIR image demonstrates minimal, if
any, abnormal signal associated with the core infarction. The T2*-weighted
image (Figure 1-5) also shows increased susceptibility in the right MCA
corresponding to the hyperdense thrombus on CT. The time-of-flight MR
angiography (Figure 1-6) demonstrates no flow-related enhancement in the
right MCA.
The mean transit time (MTT) map (Figure 1-7) from the MR perfusion-
weighted imaging demonstrates prolonged MTT throughout the right MCA
distribution which indicates markedly delayed contrast arrival to the MCA
territory compared to the contralateral side. Note that the area of core
infarction defined by the area of restricted diffusion (Figure 1-3) is much
smaller compared to the area of perfusion deficit defined by the MTT map.
This significant perfusion-diffusion mismatch shows noninfarcted tissue at
risk, also referred to as an ischemic penumbra.
The test images show an acute right MCA infarction with a significant
perfusion-diffusion mismatch. The test question focuses on indications for
intravenous administration of tissue plasminogen activator (tPA) and/or
thrombolysis or intra-arterial tPA performed by a neurointerventional
service. In 1996, based on the results of the 2-part National Institutes of
Neurological Disorders and Stroke acute stroke trial, the United States Food
and Drug Administration, approved intravenous tPA for the treatment of
acute ischemic stroke up to 3 hours after symptom onset. In 2008, data from
the European Cooperative Acute Stroke Study III (ECASS-III) were the
first from a randomized, placebo-controlled trial that demonstrated efficacy
of intravenous tPA beyond the 3-hour time window. The ECASS-III study
provided evidence for improved outcomes and did not show an increase in
deaths when expanding the window to 4.5 hours. Regardless of the
timeframe of tPA administration, patients given tPA were at increased risk
for intracranial hemorrhage when compared to patients receiving a placebo.
Intra-arterial pharmacological thrombolysis can be considered within 6
hours of the onset of symptoms. Mechanical thrombectomy can be
considered within 8 hours from symptom onset for anterior circulation
strokes. A wider window of up to 12–24 hours for feasible intervention is
accepted for posterior circulation strokes.
A hyperdense MCA represents an acute intraluminal thrombus. An
observational study by Mattle, et al, in 2008, found better outcomes in
patients with a dense MCA who were treated with intra-arterial tPA
compared to those who received intravenous tPA despite a more prolonged
treatment window in the intra-arterial group. Their results suggest the need
for a randomized controlled trial directly comparing the 2 treatment
strategies in patients with imaging evidence of large central vascular
occlusion.
Options A, B, and D are not correct.
Neither intravenous nor intra-arterial thrombolysis is contraindicated at 3
hours after ictus.
Review Question 1
 
References:
Hacke W, Kaste M, Bluhmki E, et al. Thrombolysis with alteplase 3 to 4.5 hours after acute ischemic
stroke. N Engl J Med. 2008;359:1317-1329.
Lansberg MG, Bluhmki E, Thijs VN. Efficacy and safety of tissue plasminogen activator 3 to 4.5
hours after acute ischemic stroke: a meta-analysis. Stroke. 2009;40:2438-2441.
Mattle HP, Arnold M, Georgiadis D, et al. Comparison of intraarterial and intravenous thrombolysis
for ischemic stroke with hyperdense middle cerebral artery sign. Stroke. 2008;39:379-383.
The National Institute of Neurological Disorders and Stroke rt-PA Stroke Study group. Tissue
plasminogen activator for acute ischemic stroke. N Engl J Med. 1995;333:1581-1587.
Answer 2 is D.
The noncontrast CT (Figure 2-1) and CTA (Figure 2-2) demonstrate a right
basal ganglia hypertensive hematoma with intraventricular extension. There
is blood within the left lateral ventricle as well. There is no significant
midline shift. On the axial and coronal images from the CTA (Figures 2-2
and 2-3) there are 2 foci within the hematoma that are hyperdense to the
surrounding hematoma, representing contrast extravasation. This is called
the “spot sign” and has been shown to be a predictor of hematoma
expansion. The follow-up CT (Figure 2-4) demonstrates such a hematoma
expansion, showing a significant increase in the size of the right lateral
ventricular component of the hematoma with worsening right-to-left
midline shift. A left frontal ventriculostomy catheter is also seen.
Option A is not correct.
Casting of the ventricle does not predict hematoma expansion.
Option B is not correct.
Hemorrhage in the contralateral ventricle does not predict hematoma
expansion.
Option C is not correct.
Although midline shift indicates a poor outcome, it does not predict
hematoma expansion.
Review Question 2
 
References:
Brouwers HB, Falcone GJ, McNamara KA, et al. CTA Spot sign predicts hematoma expansion in
patients with delayed presentation after intracerebral hemorrhage. Neurocrit Care. 2012;17:421-
428.
Delgado Almandoz JE, Yoo AJ, Stone MJ, et al. Systematic characterization of the computed
tomography angiography spot sign in primary intracerebral hemorrhage identifies patients at
highest risk for hematoma expansion: the spot sign score. Stroke. 2009;40:2994-3000.
Answer 3 is A.
Axial CT images (Figures 3-1 and 3-2) show a large right temporal
hematoma, intraventricular blood, and extensive subarachnoid hemorrhage.
The classic manifestation of aneurysmal rupture is subarachnoid
hemorrhage. However, about 10%–20% of cerebral aneurysm ruptures will
present with an intraparenchymal hematoma in addition to subarachnoid
hemorrhage. It is important to consider this possibility in the differential
diagnosis of intraparenchymal hematomas, as urgent treatment of the
underlying aneurysm might be lifesaving. The extensive spread of
subarachnoid blood is typical in cerebral aneurysm ruptures, and the
location of the parenchymal hematoma helps to predict the vessel of origin
of the aneurysm.
For instance, aneurysmal hemorrhage localized to the inferior frontal
lobe/gyrus rectus indicates rupture of an anterior communicating artery
aneurysm and aneurysmal hemorrhages localized to the temporal lobe point
to a middle cerebral artery aneurysm. Distal anterior cerebral artery
aneurysms present with callosal/pericallosal hemorrhage in cases of
aneurysmal rupture with parenchymal hematoma.
A worsened outcome is associated with intraparenchymal hematoma
secondary to aneurysm rupture.
Option B is not correct.
Hypertensive hematomas may occur in lobar locations, although most
commonly they are found in parts of the brain supplied by small penetrating
arteries: the basal ganglia, thalami, pons, and cerebellar white matter. These
hemorrhages can enter the ventricular system and even the subarachnoid
space, but the amount of subarachnoid blood is usually less than the amount
seen in the test case.
Option C is not correct.
Amyloid angiopathy can present with isolated subarachnoid hemorrhage,
but when this occurs, subarachnoid hemorrhage is usually limited to a few
supratentorial sulci and does not typically involve the basilar cisterns as in
the test case. Amyloid angiopathy-related lobar hematomas can bleed into
the subarachnoid space.
Option D is not correct.
Arteriovenous malformation rupture is a part of the differential diagnosis of
multicompartmental intracranial hemorrhage, but statistically, aneurysm
rupture is much more likely, especially in a patient of this age.
Review Question 3
 
Reference:
Tokuda Y, Inagawa T, Katoh Y, Kumano K, Ohbayashi N, Yoshioka H. Intracerebral hematoma in
patients with ruptured cerebral aneurysms. Surg Neurol. 1995;43:272-277.
Answer 4 is D.
The axial CT image (Figure 4-1) shows hyperdensity in the perisylvian
sulci. Note that this hyperdensity is not limited to the sulci but also involves
the adjacent cortex (Figure 4-2) because the thickness of the hyperdense
material is larger than the size of the sulci. Also the borders of the
hyperdense material are blurred as would be expected with an
intraparenchymal location. There is no significant mass effect.
The CT was performed following an endovascular procedure for
thrombolysis in a patient with an acute stroke. Extravasation of contrast
material into the subarachnoid space and/or staining of the brain
parenchyma is frequently seen, particularly after the use of mechanical
thrombolysis devices. This is secondary to leakage of contrast material
through the loose endothelial tight junctions into the extravascular space of
the injured tissue. Such extravasation can easily be confused with
intraparenchymal and/or subarachnoid hemorrhage which would suggest
vessel rupture during the procedure or hemorrhagic transformation of the
infarct, both of which are associated with negative outcome. Hyperdensity
in the brain parenchyma after thrombolysis occurs in areas of infraction.
Although the differentiation of contrast material from hemorrhage may not
always be easy, attenuations of more than 120 Hounsfield units (HU)
suggest a diagnosis of contrast material extravasation because the density of
clot rarely exceeds 100 HU. Not infrequently, blood and iodine may be
present at the same time. In difficult cases, a follow-up CT in 24 hours may
be helpful since it can demonstrate dissipation or dilution of contrast
material, whereas the hyperdensity of blood products is more persistent.
T2*-weighted gradient-echo (GRE) or susceptibility-weighted imaging
(SWI) sequences may also help differentiate contrast material from blood
products. Dual-energy CT can also discriminate between blood and iodine,
based on the differences between the photoelectric and Compton scattering
components of the x-ray attenuation of iodine and other tissue.
Option A is not correct.
The hyperdensity involves the posterior subinsular region but also spreads
in the subarachnoid space.
Option B is not correct.
The hyperdensity is ill-defined and primarily within the brain parenchyma.
This goes against an extra-axial hemorrhage.
Option C is not correct.
Cortical laminar necrosis is noted on CT as gyriform hyperdensity primarily
corresponding to calcium deposition. It will not appear in the setting of an
acute infarct. Hyperintense T1 signal secondary to laminar necrosis is
typically seen a few days after the first presentation of infarct.
Review Question 4
 
Reference:
Ghobrial GM, Nair AK, Dalyai RT, et al. Contrast stasis on noncontrast computed tomography as a
predictor of stroke postthrombolysis. Neurosurg Focus. 2011;30:E13.
Answer 5 is D.
The T2 axial FLAIR images (Figures 5-1, 5-2, 5-3, and 5-4) demonstrate
hyperintense signal involving the periventricular and subcortical white
matter with involvement of the anterior temporal lobes. These findings, if
attributable to chronic microvascular angiopathic change, are out of
proportion for the patient’s chronological age.
Additionally, there is involvement of the right and left external capsule.
Cerebral autosomal dominant arteriopathy with subcortical infarcts and
leukoencephalopathy (CADASIL), first described in 1976, is an autosomal
dominant hereditary disease caused by dominant mutations in the NOTCH3
gene. Findings more characteristic for CADASIL include involvement of
the external capsule and the anterior temporal lobe subcortical white matter.
Clinically, most patients report late onset migraine-type headaches.
Option A is not correct.
There is no cortical or basal ganglia involvement. Creutzfeldt-Jakob disease
(CJD) and bovine spongiform encephalopathy are prion-related diseases.
Findings for CJD often include cortical and basal ganglia restricted
diffusion. Variant CJD often shows bilateral involvement of the pulvinar of
the thalamus.
Option B is not correct.
Neuromyelitis optica (NMO) spares the cerebrum and involves the optic
nerve in the majority of cases. Antibodies to aquaporin 4 are characteristic
for NMO and allow differentiation of NMO from multiple sclerosis and
other demyelinating processes.
Option C is not correct.
Although there is involvement of the subcortical white matter in the
borderzone territories of patients with severe bilateral internal carotid artery
stenoses, this option is not the best answer, particularly given the patient’s
age. Ischemic disease, whether secondary to small or large vessel
arteriopathy, tends to spare the subcortical temporal white matter and
external capsule, unless it is severe.
Review Question 5
 
References:
Chabriat H, Joutel A, Dichgans M, Tournier-Lasserve E, Bousser MG. Cadasil. Lancet Neurol.
2009;8:643-653.
Singhal S, Rich P, Markus HS. The spatial distribution of MR imaging abnormalities in cerebral
autosomal dominant arteriopathy with subcortical infarcts and leukoencephalopathy and their
relationship to age and clinical features. AJNR Am J Neuroradiol. 2005;26:2481-2487.
Answer 6 is A.
The findings on this SWI sequence (Figure 6-1) demonstrate superficial
siderosis manifested by hypointensity along the cortical surfaces.
Susceptibility hypointensity representing hemosiderin may be a
consequence of recurrent subarachnoid space hemorrhage (SAH).
Intracranial hypotension is the only choice that is not associated with SAH.
Intracranial hypotension is associated with subdural hemorrhages.
Option B is not correct.
Tumors can present with SAH. Small repeated subarachnoid bleeds are
often associated with myxopapillary ependymoma of the spinal cord and
may cause superficial siderosis of the spinal cord and posterior fossa
structures. In the setting of superficial siderosis, imaging the spinal cord
must be performed when an intracranial etiology is not found.
Option C is not correct.
Superficial siderosis may result from prior aneurysm rupture. The location
may vary depending on the location of the ruptured aneurysm, but it is most
commonly seen in the basilar portions of the brain.
Option D is not correct.
Parenchymal cerebral microhemorrhages are a common MR finding of
cerebral amyloid angiopathy (CAA). However, superficial siderosis has also
been described as an additional important finding. In a small series,
superficial siderosis was seen in 60% of patients with CAA, and in few
cases, superficial siderosis was seen in the absence of parenchymal
microbleeds. Superficial siderosis localized to supratentorial sulci in an
elderly patient should raise the suspicion of CAA even in the absence of
characteristic microhemorrhages.
Option E is not correct.
Arteriovenous malformations can present with intraparenchymal and/or
subarachnoid hemorrhage, leading to superficial siderosis.
Review Question 6
 
References:
Kumar N. Neuroimaging in superficial siderosis: an in-depth look. AJNR Am J Neuroradiol.
2010;31:5-14.
Linn J, Halpin A, Demaerel P, et al. Prevalence of superficial siderosis in patients with cerebral
amyloid angiopathy. Neurology. 2010;74:1346-1350.
Answer 7 is C.
Figure 7-1 is an axial CT angiographic image of the brain showing
asymmetric prominence of the right lenticulostriate arteries and the right
superficial temporal artery, and decreased caliber of the M2 and M3
branches of the right middle cerebral artery (MCA). Moyamoya disease is
characterized by stenoses of the distal internal carotid arteries with
extensive collateralization from the lenticulostriate and thalamostriate
vessels. In the setting of moyamoya disease, narrowing of the very distal
internal carotid artery leads to diminished flow in the MCA, often the
anterior cerebral artery, and enlargement of lenticulo- and thalamostriate
vessels. Treatment by pial synangiosis involves an indirect anastomosis
from scalp vessels such as the superficial temporal artery to the MCA
territory. Over time, the superficial temporal artery hypertrophies as it
supplies the affected vascular territory.
Option A is not correct.
Serpentine dilated lenticulostriate arteries are a part of the moyamoya
disease process, but not of its treatment.
Option B is not correct.
Small MCA branches are a part of the moyamoya disease process, but not
of its treatment.
Option D is not correct.
Moyamoya disease does not directly result in ventricular dilation, and the
treatments, therefore, do not impact the size of the ventricular system.
Review Question 7
 
Reference:
Togao O, Mihara F, Yoshiura T, et al. Cerebral hemodynamics in moyamoya disease: correlation
between perfusion-weighted MR imaging and cerebral angiography. AJNR Am J Neuroradiol.
2006;27:391-397.
Answer 8 is C.
Bilateral parietal and occipital subcortical white matter T2 FLAIR
hyperintense lesions (Figures 8-2 and 8-3) are present in a symmetrical
distribution with no restricted diffusion on DWI (Figure 8-1) or abnormal
enhancement on contrast-enhanced T1-weighted (Figure 8-4) imaging.
Posterior reversible encephalopathy syndrome (PRES) is a neurotoxic
process that has relatively characteristic imaging features. The condition
occurs in a number of clinical settings including severe hypertension,
eclampsia/preeclampsia, immunosuppression (related to organ
transplantation), chemotherapy, and autoimmune disease. The mechanism is
poorly understood. Theories as to the cause include hypertension with failed
autoregulation and hyperperfusion as well as endothelial dysfunction.
On imaging (Figure 8-3, annotated), characteristic symmetric edema is
typically seen involving the posterior parietal and occipital white matter,
although frontal lobe (especially the superior frontal gyri), brainstem, and
cerebellar involvement may also be seen.
Cortical involvement often coexists in affected areas. The edema may be
appreciable by CT, although T2 FLAIR MRI sequence is most sensitive to
these changes. Restricted diffusion is generally not present, and is seen only
in unusual cases in which affected regions have undergone frank infarction.
Associated microhemorrhage may be seen on T2*-weighted GRE images.
Option A is not correct.
The lesions shown do not show restricted diffusion and, therefore, do not
represent acute multifocal infarcts. Acute infarcts predominantly involve the
cortex.
Option B is not correct.
Although, like PRES, multiple sclerosis (MS) is primarily a white matter
disease process, the symmetry and distribution of the abnormality is
unusual for MS. MS typically manifests with smaller, more discreet white
matter lesions, often in a perivenular, periventricular distribution.
Option D is not correct.
Periventricular leukomalacia occurs as a result of perinatal insult, and the
patient demographic does not fit this diagnosis. Furthermore, the volume
loss and cystic changes often seen in white matter along the ventricular
surface within the white matter are not present in the test case.
Review Question 8
 
Reference:
Bartynski WS. Posterior reversible encephalopathy syndrome, Part 1: Fundamental imaging and
clinical features. AJNR Am J Neuroradiol. 2008;29:1036-1042.
Answer 9 is B.
A rounded lesion is present in the right parieto-occipital lobe which shows
increased signal on DWI (Figure 9-1) and decreased signal on ADC (Figure
9-2) images signifying restricted diffusion. A large area of surrounding
increased diffusion is seen compatible with vasogenic edema which is better
demonstrated on the T2-weighted image (Figure 9-3) as an area of
hyperintensity. On the T2-weighted (Figure 9-3) and T1-weighted (Figure
9-4) images, the lesion consists of a large central area and a peripheral rim
that shows decreased T2 signal and marked enhancement on the contrast-
enhanced T1-weighted image (Figure 9-5).
A brain abscess is a focal, organized, encapsulated suppurative infection
within the brain parenchyma. Imaging is not specific, but in the appropriate
clinical setting, certain imaging findings can strongly suggest the diagnosis.
Brain abscesses may be solitary, but may be multiple, particularly in
immunocompromised patients. Abscesses most commonly are centered at
the gray-white interfaces. Rim enhancement is usually smooth and may be
thinner medially. Restricted diffusion, likely due to viscous debris, favors
the diagnosis of a bacterial abscess over other etiologies in the differential
diagnosis of rim-enhancing lesions. Abscess-related restricted diffusion
occurs in the central, nonenhancing part of the lesion, whereas restricted
diffusion associated with hypercellularity of malignant lesions typically
matches the enhancing rim and spares the nonenhancing parts.
Susceptibility-related hypointensity is often seen in the rim of the abscess.
The accuracy of these findings is sufficiently high to allow a specific
diagnosis in most cases of pyogenic abscesses, although nonpyogenic
abscesses can have a much wider spectrum of imaging findings that can
overlap with those of primary and secondary tumors.
Option A is not correct.
The primary differential diagnosis for a single ring-enhancing lesion in the
brain includes abscess, glioblastoma, and metastasis. A high degree of
restricted diffusion as seen in this patient is uncommon for glioblastoma
and metastases. When present, the restricted diffusion of glioblastoma or
metastases is usually seen at the enhancing part of the lesion and not in its
nonenhancing center. Perfusion imaging will show increased blood volume
in solid portions of glioblastomas versus reduced blood volume in
abscesses.
Option C is not correct.
Acute infarct will show diffusion restriction but will not show peripheral
rim enhancement. A subacute infarct, however, may show rim
enhancement. Infarcts do not typically exhibit extensive vasogenic edema as
seen on the axial T2-weighted image (Figure 9-3).
Option D is not correct.
Metastases show features similar to those of glioblastomas. Diffusion-
weighted imaging combined with perfusion-weighted imaging often allow
differentiation from pyogenic abscesses.
Review Question 9
 
Reference:
Stadnik TW, Demaerel P, Luypaert RR, et al. Imaging tutorial: differential diagnosis of bright lesions
on diffusion weighted MR images. RadioGraphics. 2003;23:e7.
Answer 10 is B.
Patchy areas of increased T2 FLAIR (Figure 10-1) and decreased T1 signal
(Figure 10-2) are present in the mid brain and right temporal subcortical
regions with contrast-enhanced T1-weighted imaging (Figure 10-3)
showing extensive leptomeningeal enhancement along the brain stem, the
suprasellar cistern, and the sylvian fissures. There are also multiple nodular
parenchymal lesions demonstrated on the contrast-enhanced T1-weighted
image (Figure 10-4, annotated).
Tuberculosis (TB) can have a varied imaging appearance, including basilar
meningeal enhancement and enhancing parenchymal lesions, or both, as in
the test case.
Tuberculosis meningitis is the most common manifestation of TB infection
in the central nervous system and usually occurs secondary to
hematogenous spread. Thick, irregular, nodular basilar meningeal
enhancement is most typical, although involvement over the cerebral
convexities can be seen concurrently. This pattern of enhancement is not
specific for TB, and can also be seen with other infectious processes,
inflammatory diseases such as sarcoidosis, and with metastatic tumor
spread (carcinomatosis).
Parenchymal TB lesions can be unifocal or multifocal, range in size from a
few millimeters to a few centimeters in diameter and show ring-like, solid
nodular, or target enhancement patterns. Lesions may show associated
calcifications and vasogenic edema, mass effect, and midline shift. Central
hypointensity and peripheral hyperintensity on T2 weighting is
characteristic, unlike pyogenic abscesses. Hydrocephalus and infarcts are
common associated complications.
Option A is not correct.
Herpes simplex virus encephalitis most commonly presents with abnormal
hyperintensity on T2 FLAIR imaging with or without diffusion restriction.
Bilateral enhancement along the mesial temporal lobes and cingulate gyrus
is also seen.
Option C is not correct.
Human immunodeficiency virus encephalopathy will manifest most
commonly with parenchymal volume loss, but periventricular white matter
changes sparing the subcortical U fibers can also be seen.
Option D is not correct.
Listeria is rare but predominantly affects the brainstem and posterior fossa
structures.
Review Question 10
 
References:
Burrill J, Williams CJ, Bain G, Conder G, Hine AL, Misra RR. Tuberculosis: a radiologic review.
RadioGraphics. 2007;27:1255-1273.
Wasay M, Kheleani BA, Moolani MK, et al. Brain CT and MRI findings in 100 consecutive patients
with intracranial tuberculoma. J Neuroimaging. 2003;13:240-247.
Answer 11 is C.
There are bilateral symmetrical areas of increased T2 signal in the globus
pallidi. Carbon monoxide toxicity tends to result in hypoxic injury to the
globi pallidi earlier than other parts of the brain.
Option A is not correct.
Near drowning typically results in a more global pattern of anoxic injury.
Option B is not correct.
Moyamoya disease may cause multifocal infarcts, often in a borderzone
distribution. Isolated globi pallidi infarctions would be unusual in
moyamoya ischemic injury.
Option D is not correct.
Diffuse axonal injury has a varied pattern; however, it often involves the
corpus callosum (especially the splenium) and the gray-white interface.
Option E is not correct.
Cardioembolic infarcts tend to be randomly scattered in an asymmetric
pattern. They are less likely to appear as isolated involvement of the deep
gray nuclei.
Review Question 11
 
Reference:
Sener RN. Acute carbon monoxide poisoning: diffusion MR imaging findings. AJNR Am J
Neuroradiol. 2003;24:1475-1477.
Answer 12 is D.
The axial T2-FLAIR (Figure 12-1) image shows discrete and confluent
hyperintense lesions in the periventricular white matter the long axes some
of which have a perpendicular orientation with respect to the long axis of
the corpus callosum. On the T1-weighted (Figure 12-2) and T2-weighted
(Figure 12-3) images, some of the lesions show cerebrospinal fluid (CSF)-
like signal indicating loss of axons, whereas other lesions show less
pronounced decreased T1 and increased T2 signal. On the contrast-
enhanced T1-weighted image (Figure 12-4) some lesions show
enhancement. The variable enhancement of these lesions, white matter
distribution, and perpendicular orientation of many of these lesions relative
to the lateral ventricles are most suggestive of multiple sclerosis (MS).
The imaging findings in MS are widely variable depending on the phase of
the plaque(s). Hyperintense lesions can be seen within the cerebral white
matter on T2-weighted imaging and may show uniform (new plaque) or
ring-like (reactivation of old plaque) contrast enhancement. Nonenhancing
lesions are older. Plaques may be oriented with their longitudinal axis
perpendicular to the walls of the lateral ventricles, in the direction
paralleling the deep white matter veins.
Another characteristic area of involvement is the junction of the septum
pellucidum and undersurface of the corpus callosum (callososeptal
interface). Such involvement is best seen on sagittal FLAIR images.
Brainstem and cerebellar involvement can also be seen. Postcontrast images
should be interpreted with caution in patients who have been recently
administered steroids. Steroids are known to stabilize the blood-brain
barrier and may result in false-negative plaque enhancement.
The combination of lesions separated in time (enhancing versus
nonenhancing) and space (multiplicity) in the appropriate clinical context
should prompt the diagnosis of MS, which is the most common
demyelinating disease process diagnosed by imaging. Finding actively
demyelinating lesions in a patient already on a given treatment regimen for
MS may indicate the need for a change in that treatment strategy.
Option A is not correct.
Although metastatic disease often involves multifocal enhancing lesions
within the brain, these typically occur primarily at the gray-white interface
rather than the deep white matter. Most metastases have significant
accompanying vasogenic edema surrounding them.
Option B is not correct.
Although posterior reversible encephalopathy syndrome (PRES) is typically
symmetric, the lesions in PRES are generally larger, more ill-defined, and
show a predilection for the parieto-occipital white matter. Ring
enhancement is not typically seen in PRES.
Option C is not correct.
With septic emboli, all lesions would be expected to enhance, and their
distribution, similar to that of metastatic disease, should predominantly
involve the gray matter-white matter interface and deep gray nuclei.
Review Question 12
 
References:
Fillipi M. Multiple sclerosis, part I: background and conventional MRI. Neuroimaging Clin N Amer.
2008;18:563-719.
Lövblad KO, Anzalone N, Dorfler A, et al. MR imaging in multiple sclerosis: review and
recommendation for current practice. AJNR Am J Neuroradiol. 2010;31:983-989.
Answer 13 is D.
Figure 13-2a (arrow) shows the loss of the normal convex superior margin
of the mid brain, the so-called “humming bird sign.” This decreased
midbrain volume is associated with progressive supranuclear palsy (PSP).
The normal midbrain volume for comparison is seen in Figure 13-2b
(arrow). PSP is a neurodegenerative disorder associated with the
accumulation of the tau protein in certain parts of the brain. It affects
approximately 1 in every 100,000 people 60 years of age or older and is far
less common than Parkinson disease (PD).
Initial symptoms are vague and nonspecific, often leading to a delayed
diagnosis. PSP is often misdiagnosed early on as PD, Alzheimer disease, or
even Creutzfeldt-Jakob disease. Memory problems and personality changes
are common. Approximately 20% of patients develop major depression.
Choreiform movements are not part of the clinical picture of PSP.
One of the diagnostic clues for PSP is the effect of the disease on vision,
which is relatively specific and causes supranuclear vertical gaze palsy.
Another clue is the typical imaging appearance of atrophy of the mid brain
with particular thinning of the tectum and tegmentum, but relative sparing
of the cerebral peduncles, leading to the “hummingbird” appearance of the
mid brain on sagittal imaging, and the “Mickey Mouse” appearance on axial
imaging. There is some overlap in symptoms with many other
neurodegenerative disorders, but distinguishing between PD and PSP can be
important, as the prognosis for PSP is much grimmer, and patients will
typically not respond to levodopa therapy.
Option A is not correct.
PSP is associated with labile moods. Initial presenting symptoms include
personality changes such as anhedonia (inability to experience pleasure), as
well as increased irritability with periods of apathy punctuated by outbursts
of anger without apparent cause.
Option B is not correct.
PSP is associated with poor eye movement control. As the disease
progresses, most affected individuals will demonstrate vertical gaze palsy
and eventually have difficulty with all voluntary eye movements.
Occasionally patients also report blurred or double vision.
Option C is not correct.
Another cardinal feature of PSP (besides the supranuclear ophthalmoplegia,
neck dystonia, parkinsonism, behavioral and cognitive impairment,
imbalance and difficulty walking, and frequent falls) is pseudobulbar palsy,
which is due to degeneration of the corticobulbar pathways in the pyramidal
tract and leads to chewing and swallowing difficulties as well as slurred
speech.
Review Question 13
 
References:
Righini A, Antonini A, De Notaris R, et al. MR imaging of the superior profile of the midbrain:
differential diagnosis between progressive supranuclear palsy and Parkinson disease. AJNR Am J
Neuroradiol. 2004;25:927-932.
Williams DR, Less AJ. Progressive supranuclear palsy: clinicopathological concepts and diagnostic
challenges. Lancet Neurol. 2009;8:270-279.
Answer 14 is C.
T1-weighted (Figures 14-1 and 14-3) images show a well-defined mass in
the pineal region which shows iso- to hypointense signal. This mass shows
marked and homogenous enhancement on contrast-enhanced T1-weighted
images (Figures 14-2 and 14-4). The mass is iso- to hypointense on T2-
weighted image (Figure 14-5) compared to the brain parenchyma.
Following gross total resection, there have been no reported cases of
recurrence of this lesion, a pineocytoma. Pineocytomas (World Health
Organization grade I) are benign, slow-growing tumors of the pineal gland,
accounting for up to one-half of pineal parenchymal neoplasms. They are
found in all age groups, but the mean age at diagnosis is 38 years. Patients
typically present with obstructive hydrocephalus or Parinaud syndrome
(paralysis of upward gaze due to compression of the superior colliculi). On
imaging, lesions are well-circumscribed, and iso- to hyperattenuating on
CT. They are hypo- to isointense on T1-weighted MR imaging, and iso- to
hyperintense on T2-weighted MR imaging. They typically demonstrate avid
and homogeneous contrast enhancement. Treatment for pineocytoma is
resection.
Option A is not correct.
Tumors of pineal cell origin such as pineoblastoma often displace the pineal
calcifications to the tumor periphery, the so-called exploded appearance.
Tumors of germ cell origin such as germinoma commonly surround the
pineal calcification, making it central (the “engulfed” appearance).
Option B is not correct.
Pineocytomas occur with equal frequency in men and women. Of interest is
the fact that germ cell tumors in the pineal region occur approximately 3
times more frequently in men, with the germinoma subtype occurring even
more commonly (10 times higher in men than in women). However, when
an intracranial germinoma is located in the suprasellar region, there is no
sex predilection.
Option D is not correct.
Radiation therapy is not used in the treatment of pineocytomas.
Review Question 14
 
References:
Osborn AG, Preece MT. Intracranial cysts: radiologic-pathologic correlation and imaging approach.
Radiology. 2006;239:650-664.
Smith AB, Rushing EJ, Smirniotopoulos JG. From the archives of the AFIP: lesions of the pineal
region: radiologic-pathologic correction. RadioGraphics. 2010;30:2001-2020.
Answer 15 is C.
The coronal T2-weighted MRI (Figure 15-1) shows a multicystic lesion in
the medial left temporal lobe. No abnormal enhancement is seen on the
gadolinium-enhanced coronal T1-weighted MRI (Figure 15-2).
Dysembryoplastic neuroepithelial tumor (DNET) is the most likely
diagnosis of the choices presented. DNET often presents as a juxtacortical
supratentorial nonenhancing lesion containing small cystic areas.
Option A is not correct.
A primitive neuroectodermal tumor is typically seen in children and will
commonly enhance and restrict diffusion.
Option B is not correct.
Hippocampal sclerosis, or mesial temporal sclerosis, does not have cystic
changes. It causes the affected structures to lose volume.
Option D is not correct.
A pilocytic astrocytoma is typically seen in children. It often has solid
enhancing components as well as nonenhancing cysts.
Option E is not correct.
Anaplastic astrocytoma and glioblastoma are high-grade lesions that
typically enhance and restrict diffusion and are infiltrative without discrete
margins.
Review Question 15
 
Reference:
Koeller KK, Dillon WP. Dysembryoplastic neuroepithelial tumors: MR appearance. AJNR Am J
Neuroradiol. 1992;13:1319-1325.
Answer 16 is C.
Marked focal thickening of the cortex (in the left middle frontal gyrus) with
lack of associated edema and/or significant mass effect are characteristic
features of focal cortical dysplasia (FCD). FCD is the most common
anatomical cause of extratemporal epilepsy in children and young adults
and is histologically differentiated into multiple subtypes. On MRI, FCDs
can be limited to the cortex, as seen in the test case, or involve the
subcortical white matter in addition to the cortex. Approximately one-third
of FCDs are not seen on MRI because of the subtlety or even absence of
imaging findings. High-resolution MRI at high-field strength is helpful in
demonstrating those subtle findings.
Option A is not correct.
Although encephalitis can present with focal cortical T2-weighted signal
increase, this is usually associated with cortical swelling rather than
thickening, which causes effacement of the sulci and even mass effect on
the surrounding structures. Clinically, encephalitis commonly presents with
an acute change in mental status in addition to seizure activity.
Option B is not correct.
Low-grade brain tumors can present with focal thickening of the cortex and
seizure activity. These are usually associated with local mass effect and
effacement of the sulci around the lesion, whereas FCD may present with
enlargement of the sulci around the lesion as seen on the axial T2-weighted
image (Figure 16-1, annotated). Occasionally, even a pathological
distinction between cortical dysplasia and a very low-grade tumor cannot be
made.
Option D is not correct.
Vascular malformations, particularly cavernous malformations, are common
lesions in patients with epilepsy. These lesions have a characteristic
peripherally decreased and centrally increased signal on both T1- and T2-
weighted images due to varying ages of contained blood by-products. No
such finding is present in the test case.
Review Question 16
 
Reference:
Colombo N, Salamon N, Raybaud C, Ozkara C, Barkovich AJ. Imaging of malformations of cortical
development. Epileptic Disord. 2009;11:194-205.
Answer 17 is A.
Otospongiosis (otherwise known as otosclerosis) is a condition of uncertain
etiology that can present with conductive, sensorineural, or mixed hearing
loss and may be unilateral or bilateral. Occasionally patients may present
with pulsatile tinnitus due to the increased marrow vascularity in affected
regions. Fenestral and cochlear (retrofenestral) otosclerosis are the 2 major
types of this condition: fenestral is more common, and cochlear most often
occurs in conjunction with fenestral otosclerosis. Fenestral otosclerosis can
result in fixation of the stapes footplate, which causes conductive hearing
loss. Stapes prostheses may be placed to help with conductive hearing loss.
Cochlear otospongiosis results in sensorineural hearing loss, and fluoride
supplementation may help slow progression of disease. In severe cases,
cochlear implantation is the only option to restore hearing.
In fenestral otospongiosis, subtle demineralization is almost always seen at
the fissula ante fenestram using high-resolution CT imaging. In cochlear
otospongiosis, radiolucent patches are present in the otic capsule
surrounding the cochlea, vestibule, and about the round window.
On MRI, subtle enhancement of otospongiosis plaques may be seen on
high-resolution postcontrast T1-weighted images. In the chronic phase of
the disease, affected regions become sclerotic and may not be detectable on
MR imaging. The demineralization involving the cochlear capsule in the
test patient is compatible with cochlear (retrofenestral) otosclerosis with
sensorineural hearing loss. Imaging findings as well as hearing loss are
almost always bilateral but may not be symmetric.
Option B is not correct.
Tympanosclerosis is a complication of chronic otitis and involves
opacification and associated calcification in the middle ear. This is not seen
in the test case.
Option C is not correct.
Acquired cholesteatoma involves opacification with associated erosive
change centered in the middle ear or external auditory canal. This is not
seen in the test case.
Option D is not correct.
In Paget disease involving the temporal bones, typical imaging findings
include a disorganized trabecular network in the medullary cavity and
cortical thickening.
Review Question 17
 
References:
Sakai O, Curtin HD, Fujita A, Kakoi H, Kitamura K. Otosclerosis: computed tomography and
magnetic resonance findings. Am J Otolaryngol. 2000;21:116-118.
Swartz JD. Sensorineural hearing deficit: a systematic approach based on imaging findings.
RadioGraphics. 1996;16:561-574.
Answer 18 is A.
Axial T2-weighted (Figure 18-1), T1-weighted (Figure 18-2), and contrast-
enhanced T1-weighted (Figure 18-3) images through the C1 vertebra level
just below the skull base show a well-defined mass in the left poststyloid
parapharyngeal space (also known as carotid space), which shows marked
enhancement. Note multiple small flow voids in the mass indicating arterial
vessels resulting in a “salt-and-pepper” appearance. The sagittal image
(Figure 18-4) shows that the mass extends into the jugular foramen. The
location of the lesion in the left jugular foramen and the clinical complaints
of the patient are suggestive of a paraganglioma, or glomus jugulare tumor.
Patients with such a diagnosis report a long period of insidious symptoms.
Paragangliomas arise from paraganglia made up of chromaffin-negative
glomus cells. These normally act as chemoreceptors along blood vessel
walls. Most paragangliomas are found in the abdomen, with a small number
occurring in the chest, and a minority (3%) occurring in the head and neck.
Very rarely are these tumors malignant.
Although many patients present with months or years of subjective
symptoms such as pain and tinnitus, occasionally they can present with
hearing loss, hoarseness, and tongue deviation due to involvement of the
cranial nerves VIII, X, and XII, respectively.
Option B is not correct.
A thrombosis would not be expected to demonstrate this degree of mass
effect or erode/expand the jugular foramen.
Option C is not correct.
The carotid artery flow void is clearly seen anteromedial to the mass.
Although it is slightly displaced and perhaps mildly narrowed, it is not
dissected.
Option D is not correct.
A schwannoma would not demonstrate internal flow voids. On CT scans, a
schwannoma typically is surrounded by sharply demarcated bony margins,
whereas a paraganglioma can cause irregular bony destruction.
Review Question 18
 
References:
Eldevik OP, Gabrielsen TO, Jacobsen EA. Imaging findings in schwannomas of the jugular foramen.
AJNR Am J Neurorad. 2000;21:1139-1144.
Van den Berg R. Imaging and management of head and neck paragangliomas. Eur Radiol.
2005;15:1310-1318.
Answer 19 is C.
Figures 19-1, 19-2, 19-3, and 19-4 reveal a left frontal sinus lesion that
expands the sinus and extends into the orbit. Only minimal rim
enhancement is present on the contrast-enhanced image (Figure 19-4)
although the lesion is hyperintense on the unenhanced T1-weighted image
(Figure 19-3).
Lack of enhancement within the lesion virtually excludes a neoplastic mass
as essentially all sinus neoplasms, either benign or malignant, will show
some solid component and enhancement. Increased T1 signal is secondary
to high protein content of the fluid filling the sinus secondary to
inflammatory obstruction of the drainage path of the sinus and/or mucus
glands in the sinus mucosa. This is a case of a mucocele that demonstrates
characteristic imaging features, including sinus wall erosion and sinus
expansion without associated central contrast enhancement.
Option A is not correct.
Homogeneous T2-weighted hyperintensity is a common feature of
inflammatory processes in the paranasal sinuses. Most malignant masses
will have only mild-to-moderate T2-weighted hyperintensity which is often
heterogeneous. Benign neoplasms will often show a higher T2 signal and
more homogeneous appearance than malignant masses.
Option B is not correct.
T1-weighted hyperintensity in a sinus mass is secondary to concentrated
protein within sinus secretions and is a good indicator of benign mucus. Be
aware that melanoma, however, as well as hemorrhagic masses, can have
T1-weighted hyperintensity.
Option D is not correct.
Bone expansion occurs secondary to slow-growing lesions, such as
mucoceles, inflammatory polyps, and benign neoplasms of the paranasal
sinuses. Most malignant masses will destroy bone and not allow enough
time for bone to remodel and expand.
Review Question 19
 
Reference:
Aygun N, Zinreich SJ. Imaging for functional endoscopic sinus surgery. Otolaryngol Clin North Am.
2006;39:403-416, vii.
Answer 20 is B.
There is a well-defined mass arising from the left inferior aspect of the
clivus and extending intracranially to the perimedullary cistern.
The bright, bubbly appearance on T2-weighted imaging (Figure 20-1) is
typical of chordomas, and is secondary to the lesion’s high fluid content.
Hemorrhage, calcification, and proteinaceous mucus can also be present and
cause some heterogeneity of signal on both T1- and T2-weighted imaging.
Most chordomas demonstrate avid enhancement, which may be
heterogeneous due to necrosis or mucinous material contained within the
tumor.
Chordomas arise from remnants of the notochord and can occur anywhere
along the skull base and vertebral column, typically in the midline.
Approximately one-third occur in the skull base related to the clivus
(spheno-occipital), one-third occur in the vertebral bodies, and one-third
occur in the sacrococcygeal region. They are locally aggressive and can
rarely metastasize. Local treatments include surgery and proton beam
radiation therapy.
Option A is not correct.
Although a metastatic lesion is possible, particularly in an adult, the
homogeneous bright T2 signal and bubbly appearance with a relatively
narrow zone of transition is less suggestive of metastatic disease.
Option C is not correct.
Cholesterol granulomas are lesions that are typically bright on both T1-and
T2-weighted imaging, which is thought to be due to hemorrhage within the
lesion. These lesions typically develop in the petrous apex, and not in the
area of abnormality shown on the test images.
Option D is not correct.
This is not a pituitary macroadenoma. Features that help differentiate
between chordomas and pituitary macroadenomas include identifying the
pituitary separately from the mass (which is typically more honeycomb-like
and with less pronounced enhancement than the chordoma), and the
morphology of the chordoma, which is typically multilobulated and can
extend dorsally into the prepontine cistern.
Review Question 20
 
References:
Erdem E, Angtuaco EC, Van Hemert R, Park JS, Al-Mefty O. Comprehensive review of intracranial
chordoma. RadioGraphics. 2003;23:995-1009.
Walcott BP, Nahed BV, Mohyeldin A, Coumans JV, Kahle KT, Ferreira MJ. Chordoma: current
concepts, management, and future directions. Lancet Oncol. 2012;13: e69-e76.
Answer 21 is C.
There is a rounded mass in the superficial lobe of the right parotid which
shows prominent T2-hyperintensity and marked contrast enhancement. The
lesion, a pleomorphic adenoma, has a distinct appearance on MRI, which
helps narrow the differential diagnosis. Although considered benign,
pleomorphic adenoma has a wide variety of histologic features and contains
epithelial and myoepithelial tissue within a “pseudocapsule.” There is a
high rate of recurrence with resection, particularly for those that are located
in proximity to the facial nerve and in the deep portion of the parotid gland.
Partial or total parotidectomy is preferred to reduce the risk of recurrence.
There is a small risk of malignant transformation of a pleomorphic adenoma
into a carcinoma, referred to as carcinoma ex pleomorphic adenoma. Risk
factors include older age, large size, recurrence, and greater age of the
lesion.
Option A is not correct.
On imaging, a distinct feature of the pleomorphic adenoma compared to
other salivary neoplasms is its well-defined margin, which is formed by the
so-called pseudocapsule. This is not an absolute barrier; microscopic
extension of the lesion beyond the margin leads to the high rate of
recurrence despite surgical resection.
Option B is not correct.
Pleomorphic adenomas are typically solitary lesions and are not often
bilateral. Warthin tumors, the second most common benign parotid gland
tumor (after pleomorphic adenoma), are typically multifocal and can be
bilateral in up to 15% of cases. Warthin tumors are associated with smoking
and radiation therapy to the neck and are more often found in older men.
Option D is not correct.
This is not the benign lymphoepithelial lesions seen in the parotid glands
that can be associated with human immunodeficiency virus infection. These
lesions are cystic, tend to be multifocal, and only demonstrate a thin rim of
enhancement. They are usually small lesions, although they can, at times,
become large.
Option E is not correct.
Pleomorphic adenomas exhibit markedly elevated and fairly homogeneous
T2-weighted signal in the great majority of cases. Moderate T2-weighted
hyperintensity is a common feature for malignant salivary gland tumors. In
patients with salivary gland mass lesions demonstrating isointense or
hypointense signal with T2 weighting, diagnoses other than pleomorphic
adenoma should be considered.
Review Question 21
 
References:
Kato H, Kanematsu M, Mizuta K, Ito Y, Hirose Y. Carcinoma ex pleomorphic adenoma of the parotid
gland: radiologic-pathologic correlation with MR imaging including diffusion-weighted imaging.
AJNR Am J Neuroradiol. 2008;29:865-867.
Yousem DM, Kraut MA, Chalian AA. Major salivary gland imaging. Radiology. 2000;216:19-29.
Answer 22 is C.
Sagittal T2 (Figure 22-1), short tau inversion recovery (STIR) (Figure 22-
2), T1 postcontrast (Figure 22-3), and axial T2 (Figure 22-4) images
demonstrate nonexpansile T2 prolongation within the lower thoracic spinal
cord and conus medullaris involving the central aspect of the cord with
preservation of the periphery of the cord. There is no associated contrast
enhancement. Due to the susceptibility associated with the spinal column,
DWI is not often performed with spine MRI. However, in the test case the
suspicion was high enough to attempt this sequence (Figure 22-5), which
demonstrated increased signal with low apparent diffusion coefficient
values consistent with acute ischemia in the clinical setting of acute onset
lower extremity weakness.
Acute spinal cord ischemia syndrome (ASCIS) is rare, accounting for
approximately 5%–8% of all acute myelopathies and 1%–2% of all vascular
neurological events. Main etiologies for ASCIS are interventions and
lesions affecting the aorta; however, this patient had no history of aortic
aneurysm or dissection.
Option A is not correct.
In the acute setting of a demyelinating plaque, such as multiple sclerosis
(MS), there would often be associated contrast enhancement. MS plaques
are usually more discrete than those of the test case, and their craniocaudal
dimension is not typically greater than 1 to 2 vertebral heights.
Option B is not correct.
Neuromyelitis optica (NMO) is an inflammatory and demyelinating
condition involving the optic nerves and the spinal cord. Clinically, optic
neuritis is commonly present when myelitis occurs. Symptoms associated
with myelitis are usually progressive in the first week of the disease onset in
contrast to ischemic cord injury which is associated with a much more rapid
onset neurologic impairment. MRI shows longitudinally extensive T2
hyperintensity which often does not spare the periphery of the cord.
Contrast enhancement of a spinal cord lesion on MRI is common in NMO.
Option D is not correct.
This is not an astrocytoma. The test patient had an acute onset. The
important distinguishing factor for astrocytomas is that they will often
present insidiously. The lack of contrast enhancement is not a distinguishing
factor as 10%–15% of astrocytomas will not enhance. Most astrocytomas
will show spinal cord expansion when they reach the size of the lesion in
the test case.
Review Question 22
 
References:
Heldner MR, Arnold M, Nedeltchev K, Gralla J, Beck J, Fischer U. Vascular diseases of the spinal
cord: a review. Curr Treat Options Neurol. 2012;14:509-520.
Mealy MA, Wingerchuk DM, Greenberg BM, Levy M. Epidemiology of neuromyelitis optica in the
United States: a multicenter analysis. Arch Neurol. 2012;69:1176-1180.
Nedeltchev K, Loher TJ, Stepper F, et al. Long-term outcome of acute spinal cord ischemia
syndrome. Stroke. 2004;35:560-565.
Answer 23 is D.
Figures 23-1, 23-2, and 23-3 demonstrate increased T2 and decreased T1
signal at the central aspects of the discovertebral junctions of the lumbar
vertebrae, indicating edema and inflammation. Figure 23-4 through the
sacrum shows ankylosis of the sacroiliac joints.
Spinal manifestations of ankylosing spondylitis include lesions of the
discovertebral complex, the costovertebral and facet joints, and the spinal
ligaments. Inflammatory lesions at the anterior and posterior vertebral body
corners are called Romanus lesions. On MRI, the inflamed marrow will
show T1-weighted hypointensity and T2/STIR hyperintensity indicating
edema or osteitis. In a later stage, when active inflammation is no longer
present, the edematous marrow will be replaced by fatty marrow and will
appear bright on T1-weighted images. These corner lesions may become
sclerotic, corresponding to the radiographic “shiny corner.” Inflammatory
involvement of the discovertebral complex (diskitis) is called the Andersson
lesion. Note the fusion at the sacroiliac joints on the axial images
supporting the diagnosis of ankylosing spondylitis.
Later focal active inflammation of the discovertebral junction (Andersson
lesion) will show T1 hypointensity and T2 hyperintensity at the central
endplate.
Edema of supporting ligaments of the spine may be seen in patients with
spondyloarthropathies, particularly at the level of the interspinous
ligaments. Arthritis of the costovertebral and facet joints may also be seen
with joint effusions, erosions, and marrow edema. Ankylosis will occur
later in the disease process. Patients with ankylosing spondylitis are at risk
for insufficiency fractures, which classically occur at the endplate/disc level.
Option A is not correct.
Discitis/osteomyelitis may also have T2-weighted hyperintense signal at the
involved intervertebral disc level accompanied by associated adjacent
endplate destructive change. Note, however, that the absence of disc
hyperintensity does not exclude discitis/osteomyelitis. The discrete
Romanus and Andersson lesions and the sacroiliac joint ankylosis are more
consistent with ankylosing spondylitis.
Option B is not correct.
There is no wedging of the vertebral bodies or kyphosis in the test case.
Scheuermann disease involves multiple lower thoracic levels with anterior
vertebral body wedging and endplate irregularity in the adolescent
population. This often results in kyphosis and scoliotic deformity. The
sacroiliac joints are spared.
Option C is not correct.
Metastatic disease will affect marrow but should not involve relatively
avascular intervertebral discs.
Review Question 23
 
Reference:
Hermann KA, Althoff CE, Schneider U, et al. Spinal changes in patients with spondyloarthritis:
comparison of MR imaging and radiographic appearances. RadioGraphics. 2005;25:559-569.
Answer 24 is B.
Figures 24-1, 24-2, 24-3, and 24-4 show an increase in the anterior-
posterior diameter of the L1 and L2 vertebrae which demonstrates increased
signal on T1- and T2-weighted images along the periphery of the vertebral
bodies and sclerotic lines paralleling the endplates on CT.
Many characteristic morphologic features of vertebral involvement in Paget
disease are present, including increased anterior-posterior dimension of the
L1 and L2 vertebral bodies, loss of concavity of the anterior vertebrae, and
thickening and hypertrophy of the trabecular bone parallel to the endplates.
These changes are related to various combinations of periosteal and
endosteal new bone formation (apposition) and endosteal bone resorption
due to increased osteoblastic and osteoclastic activity. In the late phase of
Paget disease the marrow cavity is obliterated, but in the early stages the
marrow cavity is preserved or even expanded leading to increased T1
signal, as seen in the test case. An additional characteristic feature is the
involvement of the posterior elements with the same process resulting in
marrow expansion and increased trabecular markings.
Option A is not correct.
Hemangiomas are localized lesions which present with vertical trabecular
prominence on CT and plain radiography in contrast to transverse sclerotic
lines seen in the test case. Hemangiomas are not associated with an increase
in vertebral dimension.
Option C is not correct.
Visualization of normal marrow signal (increased T1 signal) within a bone
lesion virtually excludes metastasis, with the possible exception of
melanoma metastasis (increased T1 signal from melanin).
Option D is not correct.
There is no well-defined sclerotic lesion present. The late phase of Paget
disease, in which there is marked sclerosis of the entire vertebra, can
potentially mimic osteoblastic metastasis. The morphological changes in the
vertebral body (eg, expansion of the vertebra in anterior-posterior and
transverse dimensions with lack of expansion in the craniocaudal direction
and flattening of the anterior concavity) are helpful signs in differentiating
these 2 disease processes.
Review Question 24
 
Reference:
Dell’Atti C, Cassar-Pullicino VN, Lalam RK, Tins BJ, Tyrrell PN. The spine in Paget disease.
Skeletal Radiol. 2007;36:609-626.
Answer 25 is D.
The postcontrast CT of the neck (Figures 25-1 and 25-2) demonstrates a
calcification along the caudal aspect of the anterior arch of C1, with fluid
attenuation in the prevertebral space extending caudally to the level of the
inferior endplate of C5. There is no peripheral enhancement of this fluid
collection, significant mass effect, or effacement of the airway. These
findings are typical for calcific tendinitis of the longus colli muscle. Calcific
tendinitis of the longus colli muscle is a retropharyngeal inflammatory
process that can present acutely with neck pain and pain on swallowing.
This is a benign condition that commonly resolves with conservative
management, often simply with nonsteroidal anti-inflammatory drugs.
Option A is not correct.
These findings are not compatible with an abscess. There is no mass with
peripheral enhancement. The associated calcification and edema is
pathognomonic for calcific tendinitis of the longus colli muscle. Lack of
fever may also be a clue going against abscess.
Option B is not correct.
Although neither indicated nor contraindicated, all of the findings can be
made on the contrast-enhanced CT provided. Identification of calcification
of the muscle tendon, which is essential for the diagnosis, may be
problematic on MRI.
Option C is not correct.
Although the presenting symptoms could be confused with meningismus,
for which a lumbar puncture would be considered to exclude meningitis, the
presence of the muscle tendon calcification and edema points toward the
test case diagnosis and not meningitis.
Review Question 25
 
References:
Naqshabandi AM, Srinivasan J. Teaching NeuroImages: Acute calcific tendinitis of longus colli
mimicking meningismus. Neurology. 2011;76:e81.
Offiah CE, Hall E. Acute calcific tendinitis of the longus colli muscle: spectrum of CT appearances
and anatomical correlation. Br J Radiol. 2009;82:e117-e121.
Answer 26 is D.
The late subacute phase of hematoma evolution is characterized by near-
complete conversion of deoxyhemoglobin to methemoglobin and
breakdown of the red blood cell membranes resulting in extracellular
methemoglobin. Red blood cell membrane disruption allows equalization of
intracellular and extracellular magnetic field gradients, resulting in a
decrease in susceptibility effect. Although methemoglobin is paramagnetic,
thus, expected to show decreased signal on T2-weighted images, it actually
shows increased signal on imaging sequences that are sensitive to
differences in magnetic susceptibility (GRE and SWI) and T2-weighted
images. This is secondary to multiple factors including 1) decreased T2
shortening effects from loss of the magnetic field gradient maintained by an
intact cellular membrane, 2) increased proton density and T2 signal from
increasing clotting protein concentrations, 3) extracellular water, as well as
4) shine through of markedly increased T1 signal from methemoglobin to
T2-weighted images. SWI accentuates the paramagnetic properties of blood
products, such as deoxyhemoglobin, intracellular methemoglobin, and
hemosiderin, rendering them hypointense.
Option A is not correct.
Hemosiderin (chronic phase) is paramagnetic and, thus, hypointense on
SWI and GRE sequences.
Option B is not correct.
Deoxyhemoglobin (acute phase) is paramagnetic and, thus, hypointense on
SWI and GRE sequences.
Option C is not correct.
Intracellular methemoglobin (early subacute phase) is contained within the
cell, which creates a magnetic field gradient between the intra- and
extracellular spaces, leading to susceptibility effect and hypointense signal
on SWI and GRE sequences.
Review Question 26
 
Reference:
Haacke EM, Mittal S, Wu Z, Neelavalli J, Cheng YC. Susceptibility-weighted imaging: technical
aspects and clinical applications, part 1. AJNR Am J Neuroradiol. 2009;30:19-30.
Answer 27 is B.
Cerebral microbleeds/microhemorrhages have been associated with cerebral
amyloid angiopathy and hypertensive angiopathy. They can also be seen
with ischemic stroke, Alzheimer disease, cerebral autosomal dominant
arteriopathy with subcortical infarctions and leukoencephalopathy
(CADASIL), or aging. Microhemorrhages are considered to be a marker for
small vessel disease. SWI is more sensitive than GRE in detection of
microhemorrhages. The clinical relevance of this increased sensitivity has
not been established.
Option A is not correct.
DWI is not correct because DWI is moderately sensitive for the detection of
microhemorrhages.
Option C is not correct.
GRE is not correct because it is less sensitive than SWI.
Option D is not correct.
FLAIR is not correct because it is not sensitive enough to detect
microhemorrhages.
Review Question 27
 
References:
Goos JD, van der Flier WM, Knol DL, et al. Clinical relevance of improved microbleed detection by
susceptibility-weighted magnetic resonance imaging. Stroke. 2011;42:1894-1900.
Nandigam RN, Viswanathan A, Delgado P, et al. MR imaging detection of cerebral microbleeds:
effect of susceptibility-weighted imaging, section thickness, and field strength. AJNR Am J
Neuroradiol. 2009;30:338-343.
Answer 28 is C.
Cerebral amyloid angiopathy (CAA) is a vascular disorder found in elderly
patients. CAA can be asymptomatic but typically presents with intracranial
hemorrhage. Pathologically, beta-amyloid deposits are found in the media
and adventitia of small- and medium-sized arteries, leading to degeneration
and microaneurysm formation. Manifestations of the disease include
microhemorrhages, which are occult on CT but can be identified on GRE
and SWI as small foci of low signal. These microhemorrhages tend to be
subcortical rather than located in the basal ganglia, as is typically seen with
hemorrhage associated with hypertension. CAA is not associated with
systemic amyloidosis but is commonly found in patients with Alzheimer
disease. CAA is a diagnosis of probability, or exclusion, and although it
should be considered in the test case, other causes of hemorrhage should be
excluded.
Option A is not correct.
Although hypertension can be a cause of microhemorrhage and can coexist
with CAA, the patient was normotensive upon presentation and had no
history of hypertension. This is an important differential, however, in
patients who present with intracranial hemorrhage, with or without
additional findings of extensive microhemorrhages. Chronic
microhemorrhages associated with hypertension are typically concentrated
more in the cerebral deep gray nuclei, brainstem, and cerebellum in
comparison to CAA microhemorrhages.
Option B is not correct.
Although an underlying brain tumor, either primary or metastatic, is an
important differential and should be excluded with subsequent imaging in
an elderly patient, the presence of additional microhemorrhages suggests
that the underlying cause is CAA. That said, as with hypertension, CAA can
coexist with neoplasm, and the latter should be excluded.
Option D is not correct.
The patient did not have a history of major trauma, and a large, lobar
hemorrhage would be unlikely with minor trauma unless there is an
underlying vascular malformation.
Review Question 28
 
References:
Chao CP, Kotsenas AL, Broderick DF. Cerebral amyloid angiopathy: CT and MR imaging findings.
RadioGraphics. 2006;26:1517-1531.
Haacke EM, DelProposto ZS, Chaturvedi S, et al. Imaging cerebral amyloid angiopathy with
susceptibility-weighted imaging. AJNR Am J Neuroradiol. 2007;28:316-317.
Walker DA, Broderick DF, Kotsenas AL, Rubino FA. Routine use of gradient-echo MRI to screen for
cerebral amyloid angiopathy in elderly patients. AJR Am J Roentegenol. 2004;182:1547-1550.
Answer 29 is C.
The key to answering this question is an understanding of the middle
cerebral artery (MCA) vascular distribution. The blood supply to the
thalamus is from branches of the P1 segments of the posterior cerebral
arteries, posterior communicating arteries, and medial and posterior
choroidal arteries. They are not involved in an MCA occlusion with
ischemia since the MCA does not supply the thalamus.
Option A is not correct.
A hyperdense MCA has long been established as a sign for a MCA
thrombus.
Option B is not correct.
The loss of the gray-white differentiation of the insular ribbon has also been
long established as a finding associated with early MCA infarction.
Option D is not correct.
The lenticulostriate vessels originate from the M1 segment. Loss of their
perfusion of the deep gray nuclei often results in basal ganglia
hypoattenuation.
Review Question 29
 
References:
Garg N, Eshkar N, Tanenbaum L, Cohen B, Sen S. Computed tomography angiographic correlates of
early computed tomography signs in acute ischemic stroke. J Neuroimaging. 2004;14:242-245.
Vu D, Lev MH. Noncontrast CT in acute stroke. Semin Ultrasound CT MR. 2005;26:380-386.
Answer 30 is C.
Chemosis (swelling of the conjunctiva), proptosis, and cranial bruit are
characteristic signs of carotid cavernous fistula (CCF), which is an
abnormal communication between the cavernous sinus and the carotid
arterial system. A CCF can be classified as direct or indirect. Indirect
fistulas generally develop spontaneously, and consist of communication
between the cavernous sinus and one or more meningeal branches of the
internal carotid artery (ICA), the external carotid artery, or both. Indirect
fistulas are more common in middle-aged and elderly women.
Direct CCFs result from an opening between the cavernous segment of the
ICA and the cavernous sinus as a result of trauma in approximately 80% of
the cases, while the rupture of a cavernous carotid aneurysm, fibromuscular
dysplasia, and Ehlers-Danlos syndrome may be associated with direct
CCFs. They present much more abruptly with pulsatile exophthalmos and
loss of visual acuity due to the much greater flow through the fistula
compared to that of the indirect CCF.
Option A is not correct.
The ophthalmic artery is a branch of the ICA.
Option B is not correct.
This describes a direct CCF, which usually occurs secondary to trauma,
although occasionally, the rupture of a cavernous carotid aneurysm may
present spontaneously with a direct CCF. There is no history of trauma in
the test patient. Direct CCFs are high-flow fistulas and present acutely with
symptoms that are more pronounced than that of indirect CCFs.
Option D is not correct.
Normal anastomoses exist between the branches of the external and
branches of the internal carotid arteries.
Review Question 30
 
References:
Gemmete JJ, Ansari SA, Gandhi DM. Endovascular techniques for treatment of carotid-cavernous
fistula. J Neuroophthalmol. 2009;29:62-71.
Miller NR. Dural carotid-cavernous fistulas: epidemiology, clinical presentation, and management.
Neurosurg Clin N Am. 2012;23:179-192.
Answer 31 is B.
Cavernous malformations, also called cavernomas, are composed of thin-
walled, dilated capillary beds with no intervening brain tissue.
Cavernous malformations have a distinctive appearance with a lobulated,
heterogeneously T2 hyperintense center and a dark, peripheral rim of
hemosiderin deposition (popcorn- or mulberry-like). They typically show
mild enhancement on both CT and MRI. Typically, cavernous
malformations are not associated with mass effect or surrounding edema
unless they have recently bled. These lesions can be asymptomatic;
however, depending on location and/or intermittent hemorrhage, they can
produce symptoms such as headaches, seizures, or focal neurologic deficits.
Cavernomas may be familial, in which case they are multiple. They may
occur sporadically and in such cases may be connected to a developmental
venous anomaly. Acquired cavernomas can result from radiation therapy to
the brain.
Option A is not correct.
Hyperintensity on T2-weighted imaging is frequently present in the central
portion of the lesion.
Option C is not correct.
Contrast enhancement, although mild, is often present. Subtle enhancement
is sometimes difficult to appreciate in lesions showing spontaneous T1
shortening.
Option D is not correct.
A peripheral rim of hemosiderin deposition is a typical finding in cavernous
malformations.
Review Question 31
 
References:
Brunereau L, Labauge P, Tournier-Lasserve E, Laberge S, Levy C, Houtteville JP. Familial form of
intracranial cavernous angioma: MR imaging findings in 51 families. Radiology. 2000;214:209-
216.
Jain R, Robertson PL, Gandhi D, Gujar SK, Muraszko KM, Gebarski S. Radiation-induced
cavernomas of the brain. AJNR Am J Neuroradiol. 2005;26:1158-1162.
Answer 32 is A.
Particle embolization is not typically performed on high-flow arteriovenous
malformations (AVMs) as there is no capillary bed to trap the particles, and
small particles will then pass through to the heart and then to the lungs.
Liquid embolic agents, radiosurgery, and open surgery are commonly used
to decrease flow through an AVM nidus.
Option B is not correct.
Glue embolization is the preferred method for AVM embolization.
Option C is not correct.
Radiosurgery is frequently used to obliterate AVMs.
Option D is not correct.
Open surgery can be used for the resection of AVMs.
Option E is not correct.
Balloon occlusion is also an established method. However, balloon
occlusion is not commonly performed in the United States as balloon
devices have not yet been approved for use by the United States Food and
Drug Administration.
Review Question 32
 
References:
Barr JC, Ogilvy CS. Selection of treatment modalities or observation of arteriovenous malformations.
Neurosurg Clin N Am. 2012;23:63-75.
Mossa-Basha M, Chen J, Gandhi D. Imaging of cerebral arteriovenous malformations and dural
arteriovenous fistulas. Neurosurg Clin N Am. 2012;23:27-42.
Answer 33 is B.
Pseudotumor cerebri is an idiopathic condition characterized by increased
intracranial pressure in the absence of tumor. Patients with pseudotumor
cerebri have a high incidence of bilateral high-grade stenosis in the distal
transverse sinuses. Preliminary investigations have shown improved
symptoms in patients undergoing stenting of these stenoses.
Option A is not correct.
Superior sagittal sinus has no known predilection for stenosis associated
with pseudotumor cerebri. The distal transverse sinus is the typical location.
Option C is not correct.
The straight sinus has no known predilection for stenosis in pseudotumor
cerebri. The distal transverse sinus is the typical location.
Option D is not correct.
The cavernous sinus has no known predilection for stenosis in pseudotumor
cerebri. The distal transverse sinus is the typical location.
Option E is not correct.
Internal jugular veins have no known predilection for stenosis associated
with pseudotumor cerebri. The distal transverse sinus is the typical location.
Review Question 33
 
Reference:
Ahmed RM, Wilkinson M, Parker GD, et al. Transverse sinus stenting for idiopathic intracranial
hypertension: a review of 52 patients and of model predictions. AJNR Am J Neuroradiol.
2011;32:1408-1414.
Answer 34 is B.
Because the intracranial volume is fixed, if cerebrospinal fluid volume
decreases, intracranial venous volume must increase. This concept is called
the Monro-Kellie hypothesis or doctrine. Signs of intracranial hypotension
include sagging of the mid brain below the plane of the incisura and dilation
of the dural venous sinuses. The pituitary gland may enlarge due to venous
engorgement. Pachymeningeal thickening and increased enhancement
develops and cerebellar tonsillar descent occurs. Subdural hygromas or
occasionally subdural hematomas may develop. Optic nerve sheath dilation
is a sign associated with idiopathic intracranial hypertension rather than
hypotension.
Option A is not correct.
Dilation of dural venous sinuses is a common finding in intracranial
hypotension.
Option C is not correct.
Sagging of the mid brain below the plane of the tentorial incisura is a
common finding in intracranial hypotension.
Option D is not correct.
Tonsillar herniation is a common finding in intracranial hypotension.
Option E is not correct.
Subdural hematoma can occur in intracranial hypotension.
Review Question 34
 
References:
Rohr A, Jensen U, Riedel C, et al. MR imaging of the optic nerve sheath in patients with craniospinal
hypotension. AJNR Am J Neuroradiol. 2010;31:1752-1757.
Takeuchi N, Horikoshi T, Kinouchi H, et al. Diagnostic value of the optic nerve sheath subarachnoid
space in patients with intracranial hypotension syndrome. J Neurosurg. 2012;117:372-377.
Answer 35 is B.
Imaging features of intracranial hypotension include downward sagging of
the cerebrum, cerebellum, and brainstem, engorgement of venous
structures, fullness of the pituitary gland, thickening and enhancement of
the dura, subdural effusions, and occasionally, subdural hematomas. The
dural thickening and increased contrast enhancement represents the
pachymeningeal or dural type meningeal enhancement which does not
follow the sulci. Dural (pachymeningeal) enhancement is distinguished
from the leptomeningeal enhancement pattern, which involves the
arachnoid and subarachnoid space and pia, therefore, following the sulci.
Basal meningeal enhancement as seen in tuberculosis (TB) involves the
subarachnoid space (leptomeningeal pattern). Mixed patterns are common,
as both TB and sarcoid may involve both dura and leptomeninges.
Intracranial hypotension results from cerebrospinal fluid (CSF) leak, which
may be iatrogenic, traumatic, or spontaneous. Patients typically present
with postural headaches that improve with lying down. In most cases the
dural defect causing the leak closes on its own. Occasionally, the defect
persists and intervention such as the use of an epidural blood patch may be
required to seal the dural defect.
All imaging features are not necessarily present in any given case of
intracranial hypotension, but the presence of even some of the imaging
features in the appropriate clinical setting should prompt the diagnosis.
Occasionally, imaging features not seen initially may become apparent with
follow-up imaging. Although the source of CSF leak is in the spine, MRI of
the brain with contrast is a very helpful study when this condition is
suspected clinically. MRI of the spine with contrast or CT myelography
may be required to identify the site of the CSF leak. CSF leaks from the
skull base usually present with rhinorrhea and lack many imaging features
described above, such as brain sagging and subdural collections, although
diffuse pachymeninageal enhancement is a common imaging finding in
skull base CSF leaks.
Option A is not correct.
Intracranial TB can involve brain parenchyma, dura mater, leptomeninges,
and cranial nerves. Thickened nodular leptomeningeal enhancement
centered at the skull base (basilar meningeal) can be a presentation of TB
meningitis.
Option C is not correct.
Neurosarcoid can involve any intracranial component, including brain
parenchyma, dura mater, leptomeninges, cranial nerves, and skull. Basilar
meningeal enhancement can be seen with leptomeningeal involvement by
sarcoid.
Option D is not correct.
Intracranial lymphoma can involve the brain parenchyma, dura mater,
leptomeninges, cranial nerves, and skull. Basilar meningeal enhancement
can be seen with lymphomatous involvement of the meninges.
Review Question 35
 
References:
Galassi W, Phuttharak W, Hesselink JR, Healy JF, Dietrich RB, Imbesi SG. Intracranial meningeal
disease: comparison of contrast-enhanced MR imaging with fluid-attenuated inversion-recovery
and fat-suppressed T1-weighted sequences. AJNR Am J Neuroradiol. 2005;26:553-559.
Schievink WI. Spontaneous spinal cerebrospinal fluid leaks and intracranial hypotension. JAMA.
2006;17:2286-2296.
Answer 36 is C.
Herpes simplex virus (HSV) encephalitis is one of the few infectious
processes in the brain that has relatively specific imaging findings. The
condition is associated with high morbidity and mortality. Early diagnosis
and prompt antiviral therapy can aid in producing a more favorable
outcome.
Clinically, patients present with confusion, stupor, coma, and fever. In
pediatric patients, most cases are due to a primary infection. In adults, the
majority of cases are due to reactivation of a previous infection. The latent
virus is known to reside in trigeminal ganglia and, when reactivated, can
spread along the trigeminal nerve, providing access to the anterior and
middle cranial fossae.
On imaging, sequential bilaterality in typically affected regions is highly
suggestive of HSV. On CT, findings can be subtle, but low attenuation in the
temporal lobes and insular cortex can be seen. On MRI, high T2/FLAIR
signal is seen in the mesial temporal lobes, insula, and cingulate gyrus. The
mid brain and pons may be affected, but the basal ganglia are classically
spared. Affected regions may show diffusion restriction, which implies
infarction/permanent injury. Parenchymal/meningeal enhancement may be
seen on postcontrast T1-weighted images. In later stages of the disease,
associated hemorrhage may be seen on T2*-weighted GRE sequences. Late
sequelae include encephalomalacia, atrophy, and dystrophic calcifications
of affected regions.
Option A is not correct.
Increased FLAIR signal in the mesial temporal lobe(s) is classic for HSV
encephalitis.
Option B is not correct.
Increased FLAIR signal in the cingulate region(s) is classic for HSV
encephalitis.
Option D is not correct.
Restricted diffusion in affected regions can be seen in HSV encephalitis.
The finding suggests irreversible damage in the affected regions.
Review Question 36
 
Reference:
Demaerel P, Wilms G, Robberecht W, et al. MRI of herpes simplex encephalitis. Neuroradiology.
1992;34:490-493.
Answer 37 is B.
New treatment strategies for primary brain tumors that combine surgery,
chemotherapy, and radiation therapy as well as steroids have made the
interpretation of posttreatment imaging examinations difficult.
Enhancement and edema due to necrosis, ischemia, or blood-brain barrier
changes can be misinterpreted as tumor progression. This is further
confused by the likely presence of both residual tumor and treatment
effects.
It is important to recognize potential pseudoprogression, as treatments may
be erroneously stopped if they are thought to be ineffective. Perfusion
imaging and MR spectroscopy can be helpful in distinguishing
pseudoprogression from true tumor progression, but as previously
mentioned, residual tumor and necrosis are both often present.
Option A is not correct.
The addition of antiangiogenic drugs such as bevacizumab has also
complicated posttherapy imaging analyses. However, instead of having the
appearance of growing tumor (pseudoaggression), bevacizumab can cause
the enhancing component of a tumor to shrink, while the nonenhancing
component of the tumor may continue to grow. This phenomenon is known
as pseudoresponse.
Option C is not correct.
Intrathecal methotrexate is not typically used to treat primary glial
neoplasms and has not been associated with the phenomenon of
pseudoprogression. Intrathecal methotrexate is typically used to treat
patients with leukemia and lymphoma and is associated with neurotoxicity,
both acute and long-term, affecting white matter in particular. The reason
for this toxicity is not entirely understood but may be due to metabolic
derangements leading to ischemia or demyelination.
Option D is not correct.
While undergoing steroid treatment, patients may demonstrate a decrease in
tumor enhancement and tumor-related edema.
Review Question 37
 
References:
Hygino da Cruz LC Jr, Rodriguez I, Domingues RC, Gasparetto EL, Sorensen AG.
Pseudoprogression and pseudoresponse: imaging challenges in the assessment of posttreatment
glioma. AJNR Am J Neuroradiol. 2011;32:1978-1985.
Sandoval C, Kutscher M, Jayabose S, Tenner M. Neurotoxicity of intrathecal methotrexate: MR
imaging findings. AJNR Am J Neuroradiol. 2003;24:1887-1890.
Answer 38 is D.
Primary central nervous system lymphomas (PCNSLs) usually demonstrate
no elevation in relative cerebral blood volume (rCBV) compared to the
normal brain parenchyma, whereas glioblastomas and other malignant
masses such as metastases are associated with a significant increase in
rCBV. Although often multicentric, PCNSLs can present as a solitary mass
and pose diagnostic challenges. Treatment of PCNSL is based on
chemotherapy and radiotherapy as opposed to glioblastomas for which
surgery is the first line of treatment.
Option A is not correct.
PCNSLs often occur in periventricular locations, although this also a
common feature for glioblastoma.
Option B is not correct.
Restricted diffusion is a very common feature of PCNSL, but hypercellular
tumors such as glioblastomas and metastases may also exhibit this sign.
Option C is not correct.
Most lesions that would be considered in the differential diagnosis of
PCNSL also show avid enhancement.
Option E is not correct.
Subependymal tumor spread is common for both lymphoma and
glioblastoma.
Review Question 38
 
Reference:
Schramm P, Xyda A, Klotz E, Tronnier V, Knauth M, Hartmann M. Dynamic CT perfusion imaging
of intra-axial brain tumours: differentiation of high-grade gliomas from primary CNS lymphomas.
Eur Radiol. 2010;20:2482-2490.
Answer 39 is C.
While global atrophy can be present in Alzheimer disease (AD), the degree
of ventricular enlargement is proportional to the atrophy seen globally,
except for more pronounced atrophy in the temporal and parietal lobes.
Marked ventriculomegaly in the absence of similarly enlarged sulci
suggests normal pressure hydrocephalus. Normal pressure hydrocephalus is
another structural cause for dementia, which is treated differently.
Option A is not correct.
Fluorodeoxyglucose (FDG)-PET is an emerging modality used to help with
the early diagnosis of AD as well as a way to differentiate AD from other
types of dementia, such as frontotemporal dementia. FDG-PET imaging in
patients with AD, even occasionally among those very early in the course of
their disease, will typically demonstrate hypometabolism in the temporal
and parietal lobes (the degree of which often correlates with the severity of
dementia). An early and accurate diagnosis is important both for early
intervention and intervention with the appropriate therapy, since
acetylcholinesterase inhibitors used to treat AD patients can actually
exacerbate symptoms of frontotemporal dementia.
Option B is not correct.
Global atrophy is a feature of AD but can also be seen in normal aging, and
it is highly variable. Although hippocampal and temporal lobe atrophy are
associated with AD, they are not specific findings. As with all structural and
functional imaging in dementia patients, this finding must be interpreted in
the context of clinical information that supports a specific diagnosis.
Option D is not correct.
Amyloid PET imaging is becoming available in clinical practice (although
it is not available in all geographic areas). It is also not yet reimbursed by
many insurance providers, including Medicare. Although it correlates well
with the amyloid plaque burden in AD postmortem studies, it is not yet the
“magic bullet” for diagnosis, and positive amyloid scans can be seen in
cognitively intact individuals, particularly as they age (up to 65% of people
age 80 or older). The significance of the exam results must be interpreted in
the context of other clinical information, including symptom progression,
family history, mental status examination, and neuropsychological testing.
Review Question 39
 
References:
Fleisher AS, Chen K, Liu, et al. Using positron emission tomography and florbetapir F18 to image
cortical amyloid in patients with mild cognitive impairment or dementia due to Alzheimer disease.
Arch Neurol. 2011;68:1404-1411.
Rowe CC, Ellis KA, Rimajova M, et al. Amyloid imaging results from the Australian Imaging,
Biomarkers and Lifestyle (AIBL) study of aging. Neurobiol Aging. 2010;31:1275-1283.
van de Pol LA, Korf ES, van der Flier WM, et al. Magnetic resonance imaging predictors of
cognition in mild cognitive impairment. Arch Neurol. 2007;64:1023-1028.
Answer 40 is B.
Corpus callosum involvement is not a typical feature of Wernicke
encephalopathy; however, it is seen in Marchiafava-Bignami disease, which
is an additional type of alcoholic encephalopathy. Wernicke encephalopathy
results from thiamine-deficiency, most commonly in the setting of
alcoholism.
The typical pattern of involvement includes both medial thalami, mamillary
bodies, tectal plate, and the periaqueductal gray matter. The basal ganglia,
dorsal medulla, red nuclei, cerebellum, frontal and parietal cerebral cortex
are less common sites of involvement.
Option A is not correct.
Thalami are commonly involved in Wernicke encephalopathy.
Option C is not correct.
Periaqueductal gray matter is commonly involved in Wernicke
encephalopathy.
Option D is not correct.
Mamillary bodies are commonly involved in Wernicke encephalopathy.
Option E is not correct.
Cerebellar MRI signal abnormalities, particularly of the dentate nuclei, can
occasionally be seen in Wernicke encephalopathy, but are not typical
features despite the fact that a majority of patients have cerebellar
dysfunction on neurologic exam.
Review Question 40
 
Reference:
Gallucci M, Bozzao A, Splendiani A, Masciocchi C, Passariello R. Wernicke encephalopathy: MR
findings in five patients. AJNR Am J Neuroradiol. 1990;11:887-892.
Ha ND, Weon YC, Jang JC, Kang BS, Choi SH. Spectrum of MR imaging findings in Wernicke
encephalopathy: are atypical areas of involvement only present in nonalcoholic patients? AJNR Am
J Neuroradiol. 2012;33:1398-1402.
Zuccoli G, Gallucci M, Capellades M et al. Wernicke encephalopathy: MR findings at clinical
presentastion in 26 alcoholic and nonalcoholic patients. AJNR Am J Neuroradiol. 2007;28:1328-
1331.
Answer 41 is B.
Mesial temporal sclerosis (MTS), also known as hippocampal sclerosis, is
bilateral in 10%–20% of patients with this condition. The bilateral
abnormality can be symmetric, which can be difficult to detect if the
radiologist is not familiar with the normal hippocampal morphology.
Bilateral asymmetric MTS can also occur; however, the side with a greater
degree of atrophy will not necessarily be the source of seizures.
Options A, C, D, and E are not correct.
Because Option B is correct, the other possibilities are eliminated.
Review Question 41
 
Reference:
Cheon JE, Chang KH, Kim HD, et al. MR of hippocampal sclerosis: comparison of qualitative and
quantitative assessments. AJNR Am J Neuroradiol. 1998;19:465-468.
Answer 42 is D.
A Rathke pouch forms during the fourth week of normal embryologic
development and eventually forms the anterior lobe, pars intermedia, and
pars tuberalis of the pituitary gland. This pouch normally closes in fetal
development, but a remnant can persist as a cleft that lies between the
anterior and posterior lobes of the pituitary gland. Occasionally, this
remnant enlarges to form a cyst. These are usually well-defined,
nonenhancing midline cysts within the sella turcica arising between the
anterior and posterior lobes of the pituitary. Approximately 60% are purely
intrasellar, and 40% have suprasellar extension. Rathke cysts have
imperceptible walls with no contrast enhancement demonstrated on MRI,
although in rare instances, peripheral enhancement may be present around
inflamed cysts. Intracystic hemorrhage with fluid levels has been described.
If the contents of a cyst are similar to cerebrospinal fluid (CSF) or relatively
low in protein content (less than 15%), low T1 and high T2 signal are often
seen. T1 shortening, and therefore T1 signal intensity, increases as the
protein content increases.
Option A is not correct.
Rathke cleft cysts can be hypodense or hyperdense on CT depending
mainly on protein content.
Option B is not correct.
With increases in protein content, T1 shortening will also increase, and
Rathke cleft cysts are often hyperintense to CSF on T1-weighted imaging.
Option C is not correct.
Rathke cleft cysts with low protein content will often be isointense to CSF
on all pulse sequences, and thus hyperintense on T2-weighted imaging.
Review Question 42
 
Reference:
Ross DA, Norman D, Wilson CB. Radiologic characteristics and results of surgical management of
Rathke’s cysts in 43 patients. Neurosurgery. 1992;30:173-178.
Answer 43 is C.
The abducens nerve (cranial nerve [CN] VI) innervates the lateral rectus
muscle, and thus an abducens palsy results in deficit in lateral gaze, not
medial gaze.
Palsies of CN III, IV, and VI can all result in double vision. Optic nerve
abnormalities typically result in a nonrefractive decrease in visual acuity
and can present with pain (optic neuritis).
Option A is not correct.
Pupillary dilatation (mydriasis) is seen in third nerve palsy.
Option B is not correct.
Optic neuropathy (CNII) is associated with decreased visual acuity.
Option D is not correct.
Diplopia occurs in trochlear nerve (CNIV) palsy.
Option E is not correct.
The third nerve (occulomotor) innervates the superior rectus muscle. Third
nerve damage causes upward gaze paralysis. Ptosis occurs because the third
nerve also innervates the levator palpebrae muscle.
Review Question 43
 
Reference:
Blitz A, Macedo LM, Chonka ZD, et al. Clinical high resolution CISS MRI without and with contrast
for evaluation of the upper cranial nerves: review of segmental anatomy and selected pathology of
the cisternal through extra-foraminal segments. Neuroimaging Clin North Am. In press.
Answer 44 is B.
Pulsatile tinnitus (PT) is a symptom characterized by unilateral or bilateral
ringing in the ears, with a repetitive nature synchronous with the patient’s
arterial pulse. Etiologies are varied, and the workup can involve imaging by
multiple modalities, including ultrasound, CT, MRI, and conventional
angiography before a diagnosis is made.
Aberrant course of the internal carotid artery and atypical positioning of the
jugular bulb are rare congenital causes for PT. Carotid stenosis and dural-
based arteriovenous fistulas (AVFs) are acquired conditions that can present
with PT. Skull base glomus tumors can also be an acquired cause of the
symptom. Hypervascular conditions of the skull may also cause PT.
In the pediatric population, large vestibular aqueduct syndrome is the most
common cause of sensorineural hearing loss, but tinnitus is not part of the
clinical presentation.
Option A is not correct.
Given the increased vascularity of the marrow space in patients with
otosclerosis, patients can present with PT, although conductive hearing loss
is the most common presenting symptom.
Option C is not correct.
Paget disease is a hypervascular condition of the bone and can present with
PT when temporal bones are involved.
Option D is not correct.
A dural arteriovenous fistula consists of an abnormal connection between
arteries and veins and, when it occurs at the skull base, can cause PT.
Review Question 44
 
Reference:
Vattoth S, Shah R, Cure JK. A compartment-based approach for the imaging evaluation of tinnitus.
AJNR Am J Neuroradiol. 2010;31:211-218.
Answer 45 is E.
The T1-weighted signal intensity of an intramural hematoma depends on
the age of the blood by-products. The T1-weighted signal is essentially
isointense for the first few days, and then becomes hyperintense in the
subacute stage due to conversion of deoxyhemoglobin to methemoglobin.
Hyperintense signal from methemoglobin persists for a few months and
eventually disappears. Because the high signal of an intramural thrombus
can blend with surrounding fat on T1-weighted images, fat-suppression
techniques are used to better delineate subacute hematomas, but sole
reliance on T1-weighted images in the acute phase of dissection may
decrease detection sensitivity. Other signs of dissection include focal
enlargement of the vessel segment and eccentric location of a narrowed flow
void when the vessel is not completely occluded. T2-weighted images often
show increased signal in the acute phase due to oxyhemoglobin.
Review Question 45
Options A, B, C, and D are not correct.
Because Option E is correct, the other options are eliminated.
 
References:
Kitanaka C, Tanaka J, Kuwahara M, Teraoka A. Magnetic resonance imaging study of intracranial
vertebrobasilar artery dissections. Stroke. 1994;25:571-575.
Paciaroni M, Caso V, Agnelli G. Magnetic resonance imaging, magnetic resonance and catheter
angiography for diagnosis of cervical artery dissection. Front Neurol Neurosci. 2005;20:102-118.
Answer 46 is C.
There is a subgroup of patients presenting with squamous cell
oropharyngeal carcinoma defined by the presence of human papilloma virus
(HPV) in tumor cells, predominantly HPV type 16 (HPV-16). These
patients often present at a younger age and with no history of tobacco use.
Expression of viral E6 and E7 oncoproteins that inactivate the tumor-
suppressor proteins p53 and the retinoblastoma protein, respectively, are
necessary for malignant behavior of these tumors. Multiple retrospective
analyses have reported improved prognosis for this subgroup of patients.
Both favorable sensitivity to chemotherapy and radiation have been
reported.
Option A is not correct.
The statement is true. Both prospective and retrospective analyses have
reported both increased sensitivity to radiation as well as favorable
prognosis for HPV-positive tumors compared with non-HPV related tumors.
Option B is not correct.
The statement is true, cystic lymph nodes are strongly associated with HPV-
positive oropharyngeal carcinomas.
Option D is not correct.
HPV-related tumors are not associated with a history of tobacco use.
Review Question 46
 
References:
Ang KK, Harris J, Wheeler R, et al. Human papillomavirus and survival of patients with
oropharyngeal cancer. N Engl J Med. 2010;363:24-35.
Goldenberg D, Begum S, Westra WH, et al. Cystic lymph node metastasis in patients with head and
neck cancer: an HPV-associated phenomenon. Head Neck. 2008;30:898-903.
Answer 47 is B.
The patient has subacute combined degeneration, a degenerative process
involving the spinal cord, caused by B12 deficiency, which also causes a
megaloblastic anemia. The term combined refers to the involvement of the
dorsal columns of the spinal cord (ascending sensory tracts for fine touch,
vibration, and position) and the lateral columns (corticospinal and lateral
spinothalamic tracts), which are involved later in the course. MR findings
include T2 prolongation in the dorsal columns of the cervical and thoracic
spinal cord with mild enhancement. Pathologically, these areas demonstrate
demyelination and vacuolization. Improvement in the imaging appearance
can be seen with treatment. Other pathologic processes that can cause this
MR appearance include human immunodeficiency virus vacuolar
myelopathy, tabes dorsalis with neurosyphilis, and other nutritional
deficiencies including copper.
Option A is not correct.
T2 hyperintensity in the ventral horns would indicate a process involving
the cell bodies of the lower motor neurons, which populate this part of the
spinal cord. A process that involves the ventral horns, such as polio, would
cause flaccid paralysis in the parts of the body innervated by the affected
segments.
Option C is not correct.
Enlargement of the central spinal canal can be idiopathic, associated with
developmental anomalies such as Chiari I malformation, or obstruction of
normal cerebrospinal fluid flow due to mass. It is not associated with B12
deficiency.
Option D is not correct.
B12 deficiency is not subacutely associated with spinal cord atrophy. Over
time, damage can become permanent and cause myelomalacia of the
affected portions of the spinal cord.
Review Question 47
 
References:
Naidich M, Ho S. Case 87: subacute combined degeneration. Radiology. 2005;237:101-103.
Ravina B, Loevner L, Bank W. Subacute combined degeneration of the spinal cord: a case of
reversible cervical myelopathy. AJR Am J Roentgenol. 2000;174:863-865.
Answer 48 is A.
The clay shoveler’s fracture is an avulsion fracture of the lower cervical
spinous processes, with the mechanism of injury thought to be secondary to
muscle pull force transmission through the supraspinous ligaments. These
fractures are mechanically and neurologically stable. It is important to
differentiate clay shoveler’s fractures from those occurring secondary to
impaction of the spinous process by the adjacent spinous process as seen in
extension-type injuries, which may be a component of an unstable injury.
The most widely used model for addressing the stability of spinal fractures
is the 3 column model of Denis. This model, originally applied to the
thoracolumbar spine, is also useful in thinking about the cervical spine.
The model divides the spinal column into anterior, middle, and posterior
columns. The anterior column consists of the anterior vertebral body, the
anterior longitudinal ligament, and the anterior annulus fibrosis. The middle
column consists of the posterior vertebral body, posterior longitudinal
ligament, and posterior annulus fibrosis. The posterior column consists of
the posterior bony elements, including the pedicles, lamina, facets, and
spinous processes; ligaments, including the ligamentum flavum,
interspinous, and supraspinous; and the facet joint capsule. When only 1
column is disrupted, the injury is considered mechanically stable. When 2
or all 3 columns are disrupted, the injury is considered unstable.
Option B is not correct.
Flexion teardrop fracture is the most severe cervical spine injury with a
severe flexion force resulting in a fracture dislocation of the cervical spine.
All 3 columns are disrupted and it is commonly associated with anterior
cord syndrome.
Option C is not correct.
Isolated articular pillar refers to fracture of the pedicle and ipsilateral
lamina, allowing the pillar to rotate. These can be bilateral or unilateral and
are always considered to be unstable. Injury to the vertebral artery is
common.
Option D is not correct.
Hangman’s fracture involves fracture through the pars interarticularis of the
C2 vertebral body and is unstable.
Review Question 48
 
References:
Cancelmo JJ Jr. Clay shoveler’s fracture. A helpful diagnostic sign. Am J Roentgenol Radium Ther
Nucl Med. 1972;115:540-543.
Denis F. The three column spine and its significance in the classification of acute thoracolumbar
spinal injuries. Spine. 1988;8;817-831.
Kim KS, Chen HH, Russell EJ, Rogers LF. Flexion teardrop fracture of the cervical spine:
radiographic characteristics. AJR Am J Roentgenol. 1989;152:319-326.
Looby S, Flanders A. Spine trauma. Radiol Clin North Am. 2011;49:129-163.
Maiman DJ, Yoganandan N. Biomechanics of cervical spine trauma. Clin Neurosurg. 1991;37:543-
570.
Answer 49 is C.
Distinguishing early discitis/osteomyelitis from degenerative change can be
challenging, as both conditions can show loss of intervertebral disc height,
increased hyperintensity on T2-weighted signal in the intervertebral disc
space, and endplate marrow edema. Although the clinical context may help
narrow the differential diagnosis, some of the clinical features of the
conditions overlap, creating a diagnostic dilemma. In these cases, disc
and/or marrow biopsy may be performed to make the final diagnosis.
However, the false-negative biopsy rate is quite high. Endplate erosive
changes, best seen on T1-weighted MRI, favor infectious etiology.
However, when enhancing perivertebral and/or epidural tissue is present in
association with these findings, the diagnosis of discitis/osteomyelitis can
be made with greater confidence. In such cases, biopsy may still be
necessary to tailor the medication regimen appropriately. In cases where
there is significant canal compromise and/or cord compression, surgical
intervention may be required.
Option A is not correct.
T2 hyperintensity within a disc may be seen in both discitis/osteomyelitis
and degenerative change. Typically, disc degeneration is associated with
decreased T2 signal due to disc desiccation. Although increased disc T2
signal in the setting of disc degeneration is rare, it is still frequently seen
given the high prevalence of disc degeneration to render it not useful in
differentiating disc degeneration from infection.
Option B is not correct.
Endplate marrow edema is a common imaging feature for both
discitis/osteomyelitis and degenerative change.
Option D is not correct.
Disc height loss is a common imaging feature for both discitis/osteomyelitis
and degenerative change.
Review Question 49
 
References:
Dunbar JA, Sandoe JA, Rao AS, Crimmins DW, Baig W, Rankine JJ. The MRI appearances of early
vertebral osteomyelitis and discitis. Clin Radiol. 2010;65:974-981.
Kowalski TJ, Layton KF, Berbari EF, et al. Follow-up MR imaging in patients with pyogenic spine
infections: lack of correlation with clinical features. AJNR Am J Neuroradiol. 2007;28:693-699.
Ledermann HP, Schweitzer ME, Morrison WB, Carrino JA. MR imaging findings in spinal
infections: rules or myths? Radiology. 2003;228:506-514.
Answer 50 is A.
Vertebral endplate and subendplate marrow changes commonly occur
secondary to disc degeneration. The initial changes, called Modic type I,
have low signal on T1-weighted and high signal on T2-weighted scans.
These imaging findings correspond to marrow edema and fibrovascular
changes. These changes may evolve to fatty replacement of marrow edema
and cellular elements, called Modic type II, unless there is ongoing
mechanical instability. These fatty changes have high signal on T1- and T2-
weighted scans, unless fat suppression is used. In some cases, chronic
degeneration leads to osseous sclerosis, which has a T1 and T2 hypointense
appearance, a stage known as Modic type III degenerative changes.
Option B is not correct.
Marrow edema/inflammation is the first step.
Option C is not correct.
Marrow edema/inflammation occurs first (Modic type I) followed by fatty
changes (Modic Type II) and lastly sclerosis (Modic Type III).
Option D is not correct.
Marrow edema/inflammation is the first step.
Review Question 50
 
Reference:
Rahme R, Moussa R. The modic vertebral endplate and marrow changes: pathologic significance and
relation to low back pain and segmental instability of the lumbar spine. AJNR Am J Neuroradiol.
2008;29:838-842.

You might also like